Health Assessment Exam 1 (Review) PrepU

Pataasin ang iyong marka sa homework at exams ngayon gamit ang Quizwiz!

A 55-year-old client is being evaluated for a suspected hearing impairment. Which of the nurse's health interview questions is most likely to yield relevant data?

"Are you having difficulty hearing high-frequency sounds?" Explanation: Asking the client about changes in hearing ability with different frequency sounds would be most appropriate. This is because the client is over age 50 and may be experiencing presbycusis, a loss of ability to hear high-frequency sounds. Asking about drainage would provide information about a possible infection; asking about pain would provide information about possible ear infection, cerumen blockage, sinus infections, or teeth and gum problems. Asking about a popping sensation may be appropriate if otitis media and perforation are suspected.

The nurse is assessing cranial nerves III, IV, and VI. Which instructions should the nurse provide to the client in order to perform this assessment?

"Follow my finger with only your eyes." Explanation: Testing cranial nerves III, IV, and VI also tests the movement of the eye muscles by asking the client to move the eyes in different directions. Turning the head assesses neck range of motion and mobility. Shrugging shoulder against resistance assesses a different cranial nerve. Asking the client to stand still with the eyes closed is known as the Romberg's test to test balance.

The nurse notes thrush on the palate of a client. The most appropriate question the nurse should ask is

"Have you been on antibiotics recently?"

During the health history inquiry about alcohol intake, which of the following is a CAGE question?

"Have you ever felt annoyed by criticism about drinking?" Explanation: "Have you ever felt annoyed by criticism about drinking?" is one of the four questions that make up the CAGE questionnaire.

A client reports feeling depressed for several months since being fired from a long-term job. Which question should the nurse include when assessing this client?

"Have you thought of hurting yourself?" Explanation: If the client seems depressed, ask about thoughts of suicide: "Have you ever thought about hurting yourself or ending your life?" The severity of the depression needs to be assessed since it could be lethal. Asking about a job search or finances are not appropriate questions when a client is depressed.

A client states reports feeling like a burden to the family and totally worthless. Which response would be appropriate for the nurse to make to this client?

"Have you thought of killing yourself?" Explanation: Feelings of worthlessness and being a burden could indicate that the client is depressed and at risk for suicide. The nurse should ask if the client has ever thought of suicide. Asking about the family does not address the client's statement. Stating that the client isn't worthless or that everyone feels that way now and then minimizes the client's statement and does not assess if the client is experiencing suicide ideation.

A client is diagnosed with a scotoma. What question is appropriate for the nurse to ask to obtain more data about this condition?

"Are the blind spots constant or intermittent?" Explanation: A scotoma is the presence of blind spots that can be constant or intermittent. If they are constant it may indicate retinal detachment. Intermittent blind spots may be due to vascular spasm or pressure on the optic nerve. Floaters are a common finding in individuals with myopia or in person over the age of 40 years and are a sign of normal aging. Redness or tearing is associated with allergies or inflammation of the eye. Night blindness is associated with optic nerve atrophy, glaucoma, or vitamin A deficiency.

During an eye assessment, the nurse is testing a client's visual acuity using a Snellen chart. In order to prepare the client for this component of assessment, what instruction should the nurse provide?

"Cover one of your eyes and then read out the letters on the chart, starting from the top." Explanation: Using a Snellen chart requires that the client stand a specific distance from the chart, usually 20 feet. The client does not move, and each eye is assessed individually.

The nurse is using the COLDSPA mnemonic to assess a client's history of chest pain. What interview question addresses the "A" in this assessment model?

"Do you have any other symptoms together with your chest pain, such as nausea, sweating?" Explanation: The "A" in COLDSPA addresses associated factors, such as dyspnea, diaphoresis, pale clammy skin, nausea, and vomiting.

A client describes headaches as severe and lasting for days. Which question would be most appropriate to use to determine if these headaches are migraines?

"Do you have any visual changes before the headache?" Explanation: A typical migraine headache has prodromal symptoms that may include visual disturbances, vertigo, tinnitus, and/or numbness or tingling of the fingers and toes. <wbr />Asking about being tense or anxious would be appropriate to assess for a tension headache. Asking about alcohol or tearing would be appropriate for a cluster headache.

The nurse is assessing a middle-aged female client who is new to the clinic. The nurse observes the presence of significant facial hair that is uncharacteristic of the client's ethnicity. What assessment question should the nurse ask?

"Do you take steroid medications on a regular basis?" Explanation: Steroid therapy causes hirsutism. Dietary supplements, diabetes, and skin cancer are unlikely causes of abnormal hair growth.

A nurse is working with a client who has a history of headaches. When preparing to assess the client's temporomandibular joint (TMJ), the nurse should provide what instruction?

"I'm going to put my fingers in front of your ears and ask you to open your mouth wide." Explanation: To assess the TMJ, place your index finger over the front of each ear as you ask the client to open her mouth. None of the other listed instructions facilitates this assessment.

Which Glasgow Coma Score indicates the client is in a deep coma?

3 Explanation: A Glasgow Coma Scale score indicates the client is in a deep coma. All other scores indicate some impairment with a score of 15 being no impairment.

The nurse has completed the initial assessment of a client and is now performing data analysis. The nurse obtained a blood pressure reading of 114/70 mm Hg. What is this client's pulse pressure?

44 mm Hg Explanation: A client's pulse pressure is the difference between systolic and diastolic pressure.

A client diagnosed with Sjogren syndrome should be given which instructions?

Eye drops and sucking on hard candy may used to relieve dryness. Explanation: Sjogren syndrome is a chronic inflammatory disorder characterized by decreased lacrimal and salivary gland secretion. Eye drops and hard candy can provide relief from dryness. Sjogren syndrome does not affect blood pressure. Sjogren syndrome is not contagious or sexually transmitted. Taking mucus thinning medication does not provide relief but could actually lead to additional dryness.

The nurse assesses a client using the Glasgow Coma Scale. Which of the following indicators will be used to determine the score?

Eye opening, and appropriateness of verbal and motor responses. Explanation: The Glasgow Coma Scale rates responses to eye opening, verbal, and motor responses.

What nursing diagnosis would be most appropriate for a client admitted with heart failure?

Ineffective tissue perfusion Explanation: Heart failure can cause ineffective tissue perfusion which can lead to fatigue, pain and activity intolerance. Impaired gas exchange would be more appropriate for respiratory disorders

The nurse assesses the client to have a Glasgow Coma score of 15. The nurse anticipates what degree of impairment?

None Explanation: A Glasgow Coma Score of 15 would indicate no impairments. All other scores indicate some degree of impairment up to and including deep coma.

A nurse has completed the assessment of a client's direct pupillary response and is now assessing consensual response. This aspect of assessment should include which action?

Observing the eye's reaction when a light is shone into the opposite eye Explanation: The nurse assesses consensual response at the same time as direct response by shining a light obliquely into one eye and observing the pupillary reaction in the opposite eye. This does not involve a comparison between maximum and minimum pupil size, however. Neither eye is covered, and peripheral vision is not relevant to this assessment.

The nurse is preparing to perform the Rinne test on a client. The nurse would place the tuning fork at which location first?

On the mastoid process Explanation: For the Rinne test, the tuning fork base is place on the client's mastoid process and then it is moved to the front of the external auditory canal when the client no longer hears the sound. The tuning fork is place in the center of the client's forehead or head for the Weber test.

Which instruction to the client will help facilitate examination of the temporomandibular joint by the nurse?

Open the mouth Explanation: While performing the assessment of the temporomandibular joint, the nurse should ask the client to open the mouth. This gives an easy access to the joint. Telling the client to sit upright and not move helps in performing the overall examination; however, it does not contribute to the examination of the temporomandibular joint. Telling the client to perform a chewing action is not appropriate.

A nursing educator is evaluating a colleague's examination of a client's thyroid gland. The educator would determine that the nurse needs additional instruction when the nurse demonstrates which technique?

Percussion Explanation: When examining the thyroid gland, the nurse inspects for enlargement and asymmetry; auscultates for bruits; and palpates for tumors, masses, size, and tenderness. Percussion does not provide meaningful data.

On a health history, a client reports no visual disturbances, last eye exam two years ago, and does not wear glasses. The nurse notices that the client squints when signing the consent for treatment form and holds the paper close to the face. What should the nurse do next?

Perform both the distant and near visual acuity tests Explanation: The first thing the nurse should do is perform both the distant and near visual acuity exams to assess for loss of far and near vision. Testing the pupil is important to assess reaction to light. The findings must be documented in the client's record. If abnormalities are found upon assessment, the client should be referred for a complete eye examination.

Which part of the nursing process includes the formulation of goals?

Planning Explanation: Planning is determining outcome criteria and developing a plan. Diagnosis occurs when the data has been analyzed and a professional judgement occurs. Assessment is the collection of data. Evaluation assesses whether the outcome criteria have been met.

When assessing a client's respirations, what is most important to include in the documentation?

Presence of dyspnea Explanation: The presence of dyspnea is the most important of the choices listed to include in the documentation. Dyspnea can be an indicator of potential respiratory distress. The presence of pain and position of the client can impact the client's respiratory status, but are not the primary piece of information to include in the documentation. Assessment of pedal pulses is a component of a circulatory assessment.

A client with an inability to read billboards while driving arrives at the health care facility for an eye examination. Which piece of equipment should the nurse use to check the client's distant vision?

Snellen chart Explanation: To check the client's distant vision the nurse should use the Snellen chart. An ophthalmoscope is used to view the red reflex and examine the retina of the eye. An opaque card is used to test for strabismus. A penlight is used to test pupillary constriction.

When assessing a client with Graves disease, how would you expect the thyroid gland to be?

Soft Explanation: Soft in Graves disease; firm in Hashimoto thyroiditis, malignancy. Benign and malignant nodules, tenderness in thyroiditis.

What information aids the nurse in assessing possible biases in the data collected in the health history?

Source of information Explanation: Designating the source helps the nurse and reader assess the type of information provided and possible biases.

What is used to gauge central and peripheral nervous system disorders?

Strength of a reflex

An elderly client comes to the clinic for evaluation. During the skin assessment, the nurse notes considerable skin tenting. Why does this finding require further assessment?

Tenting indicates dehydration Explanation: A persistent pinch, or tenting of the skin, indicates dehydration. Tenting would not be present in the condition of malnutrition alone, dramatic weight loss, or vitamin B12 deficiency.

The nurse recognizes that a barrier to successful pain management for the client with opioid tolerance is:

The client does not experience pain relief with usual doses of opioids. Explanation: A barrier to successful pain management for the client with opioid tolerance is that the client does not experience pain relief with usual doses of opioids. The client with opioid tolerance has an altered physiologic response to painful stimuli, and repeated use of opioids causes their bodies to become more sensitive to pain. Pain assessment tools appropriate for use with the client with opioid tolerance exist.

The nurse is preparing to assess a client's visual fields to evaluate her gross peripheral vision. Which test would the nurse perform?

The confrontation test evaluates peripheral vision. The cover test, corneal light reflex test, and eye position test would be used to evaluate extraocular muscle function.

Which describes the nurse using the technique of percussion?

The nurse notes resonance over the individual's thorax. Explanation: The nurse uses the technique of percussion to produce sounds over various parts of the body. The nurse detects resonance over the lungs by percussing the thorax. Inspection involves smelling for odors and conscious observation of the client's physical characteristics and behaviors, such as noting symmetry of the thorax. The nurse uses palpation to detect crepitus over the thorax by the use of touch. Auscultation is used by the nurse to assess lung sounds, such as rustling.

A client reports pain as being 7 on a scale from 1 to 10. In which area of the symptom should the nurse document this information?

The seven attributes of a symptom should be assessed. The mnemonic OLD CART is used to ensure are all areas are included. Pain is documented under characteristic of the symptom. Onset identifies when the symptom began. Location is the body area including any radiation. Duration is the length of time the symptom lasts.

The nurse is taking a comprehensive health history on a new client. Why would it be essential for the nurse to obtain a complete description of the present illness?

To establish an accurate diagnosis Explanation: A complete description of the present illness is essential to an accurate diagnosis.

A nurse is assessing a 49-year-old client who questions the nurse's need to know about sunburns he experienced as a child. How should the nurse best explain the rationale for this subjective assessment?

"Having bad sunburns when you're a child puts you at risk for skin cancer later in life." Explanation: Experiencing severe sunburns as a child is a risk factor for skin cancer. The nurse is not directly assessing the client's pattern of moles in this way, nor the skin's ability to heal. The nurse is not assessing the parents' care of their child's overall skin health by asking this question.

When assessing the client's ability to make sound judgments, what question should the nurse ask?

"How do you plan to pay rent if you lose your job?" Explanation: The nurse can usually assess judgment by noting the client's responses to family situations, jobs, use of money, and interpersonal conflicts. Asking if the client eats breakfast or can manage money are simple yes/no questions that are less likely to reveal data than asking for the client's plan of action in a hypothetical situation such as job loss. Asking how many dimes are in a dollar is a knowledge question.

A client who takes oral contraceptives states that she often experiences breast pain just before her menstrual cycle begins. When using the COLDSPA mnemonic to assess the client's pain, the nurse should begin by asking which of the following?

"How would you describe your pain? Is it sharp? Is it an ache?" Explanation: The "C" in COLDSPA elicits the character of the client's pain. C-character O- onset L- location D-duration S- severity P- Pattern A- Associated factors

The nurse is obtaining information about a client's past health history. Which client statement would best reflect this aspect?

"I had surgery 5 years ago to repair an inguinal hernia." Explanation: The past health history focuses on questions related to the client's past from the earliest beginnings to the present. The statement about surgery would apply to this portion of the assessment. The statement about the parents and siblings would apply to the family health history. The statement about pain in urination would apply to the reason for seeking health care.

As part of a class assignment, each student is to ask another student questions to determine that student's critical thinking skills. Which of the following responses would reflect that the student has some critical thinking skills?

"I try to think about alternatives before making a decision." Explanation: Critical thinkers need to explore and consider other alternatives before making a decision. The final opinion or judgment is reserved until additional information is collected and all of the information is available. Critical thinkers also need to be open to the fact that they may not always be right and to use past knowledge and experience to analyze data.

A client asks why a health assessment needs to be done. What should the nurse respond to this client?

"It determines your health status, risk factors and educational needs to develop a plan of care." Explanation: The purpose of the nursing health assessment is to determine a client's health status, risk factors, and need for education as a basis for developing a nursing plan of care. It is not used as a tool to evaluate care. It is not completed to reduce the work load for the health care provider or to validate the findings from the health care provider's examination.

A 52-year-old client with myopia calls the ophthalmology clinic very upset. She tells the nurse, "I keep seeing semi-clear spots floating across my vision. What is wrong with me?" What would be the most appropriate response by the nurse?

"It is not an uncommon finding in people older than 40 years for this to happen. They are called 'floaters'." Explanation: Floaters (translucent specks that drift across the visual field) are common in people older than 40 years of age and nearsighted clients; no additional follow-up is needed.

The mother of a small child with tubes in both eardrums asks the nurse if it is okay if the child travels by airplane. What is the nurse's best response?

"It's safe to fly because the tubes will equalize pressure." Explanation: Pressure equalization tubes equalize pressure on either sides of the eardrum; so it's a great time to fly if one has tubes in the ears. The child should wear ear plugs to keep water out of the ears when swimming. Wearing ear plugs while flying may diminish the pressure equalization advantage of the tubes. Clients do not have to avoid flying for any period of time after tube placement. Ear tubes do not have an effect on immunocompromised clients.

A nursing instructor is teaching students how to assess a client's pain. The instructor emphasizes that there are many misconceptions about pain. The instructor realizes that a student needs further direction when the student states:

"Nurses are the best authority on pain." Explanation: Pain is what the client says it is, and it exists whenever the client says it does. The client is the best authority on pain, and self-report is the gold standard. Therefore, nurses are not authorities on pain. It is true that clients with chronic illnesses can and often do have chronic pain. It also is true that acute pain can be intense. Chronic pain is sometimes known as persistent pain.

An older client is concerned about new senile keratoses appearing on the skin. What should the nurse respond to this client's concern?

"These are considered a normal age-related change in the skin." Explanation: Older clients may have skin lesions associated with aging which include senile keratoses. These skin lesions are not considered skin cancer. They do not need to be cleansed and bandaged. They are not treated with medication.

A client asks why gloves are being worn during the physical examination. What should the nurse respond to this client?

"They make sure that any microorganisms on my hands do not touch your skin." Explanation: One reason to wear gloves is to prevent the transmission of flora from health care workers to clients. Wearing gloves is more than just following a policy. Gloves hinder the ability to discern body parts and positions. Although the client may have a communicable illness, the nurse should not make a statement that could cause the client anxiety about being ill.

A 6-year-old boy has come to the clinic with his mother because of recent eye redness and discharge. The nurse's assessment has suggested a diagnosis of conjunctivitis. What should the nurse tell the mother about her son's eye?

"This might have been the result of an allergy, but most likely it was caused by a bacteria or virus." Explanation: Conjunctivitis usually has an infectious etiology. Severe pain and vision damage are not common consequences.

A client has presented for care with complaints of persistent lower back pain. When assessing the client's pain, which statement, made by the nurse, would be most appropriate?

"What makes your pain better or worse?" Explanation: The nurse would assess the client's pain pattern by asking what makes a sign or symptom better or worse. A client's level of pain is subjective and the nurse would not question this level. Pain is not hereditary. The nurse would not state what makes their pain better, but rather focus on assessing the client's pain.

A client has sought care at the clinic, telling the nurse, "This ringing in my ears has gone on for weeks, and it's driving me crazy." The client denies exposure to excessive noise levels. What would the nurse ask next?

"What medications are you currently taking?" Explanation: Tinnitus may be associated with certain ototoxic medications. There is not usually a family history of this problem. Cerumen buildup can contribute, but hygiene is not a common etiology of tinnitus. The client's overall perception of health is important but is less likely to explain why he is experiencing tinnitus.

The nurse assesses the client's pulses to be normal. These would be documented how?

2+ Explanation: Normal pulses are 2+. Absent pulses are 0. Weak pules are 1+. Increased pulses are 3+.

Which vision acuity reading indicates blindness?

20/200 Explanation: The reading of 20/200 on a vision acuity test indicates blindness. The reading of 20/20 is considered normal vision. This means that the client being tested can distinguish what a person with normal vision can distinguish from 20 feet away. The top or first number is always 20, indicating the distance from the client to the chart. The bottom or second number refers to the last full line the client could read. The higher the second number, the poorer the vision. 20/40 and 20/100 also denote poor vision.

A 72-year-old teacher comes to a skilled nursing facility for rehabilitation after being in the hospital for 6 weeks. She was treated for sepsis and respiratory failure and had to be on a ventilator for 3 weeks. The nurse is completing an initial assessment and evaluating the client's skin condition. On her sacrum there is full-thickness skin loss that is 5 cm in diameter with damage to the subcutaneous tissue. The underlying muscle is not affected. What is the stage of this pressure ulcer?

3 Explanation: A stage III ulcer is a full-thickness skin loss with damage to or necrosis of subcutaneous tissue that may extend to, but not through, the underlying muscle.

While discussing family history with a client who is healthy except for a current UTI requiring IV antibiotics, the client tells the nurse that he has three sisters and two brothers. Two of his sisters have died and one brother is in a nursing home after a stroke. The nurse would include the sibling group in a genogram in what manner?

3 circles and 3 squares with lines through 2 circles

The nurse is assessing a client's tonsils and note that they touch the uvula. The nurse would document this finding as which of the following?

3+ Explanation: Tonsils that touch the uvula are identified as 3+. Tonsils that are visible are graded as 1+; midway between tonsillar pillars and uvula as 2+; touch each other as 4+.

The nurse measures a client's pupils and documents the size. Which size would the nurse document as normal?

4 mm Explanation: Pupils are normally equal in size and range from 3 to 5 mm. Size outside this range are considered abnormal.

A client is diagnosed with a stage III pressure ulcer. Which diagram should the nurse use when teaching the client and family about this skin lesion?

A Stage III pressure ulcer has full-thickness tissue loss. Subcutaneous fat may be visible but bone, tendon, or muscle is not exposed. A Stage I pressure ulcer has intact skin with non-blanchable redness of a localized area usually over a bony prominence. A Stage II pressure ulcer is a partial thickness loss of dermis presenting as a shallow open ulcer with a red-pink wound bed, without slough. This wound may also present as an intact or open/ruptured, serum-filled blister. A Stage IV pressure ulcer has full-thickness tissue loss with exposed bone, tendon, or muscle. Slough or eschar may be present on some parts of the wound bed.

Which observation would cause the nurse to suspect an abusive situation? Select all that apply.

A child is persistent in trying to please a parent. A caregiver of a cognitively intact older adult dominates the interview. A preschooler rubs her perineum and complains of it hurting. Explanation: Observations suggestive of possible abuse include a caregiver of a cognitively intact older adult dominating the interview, a child being persistent in trying to please a parent, and a preschooler rubbing her perineum and complaining of it hurting. Observations not suggestive of abuse include a parent allowing an adolescent to speak privately with the nurse and an explanation that is appropriate for an injury.

Which of the following assessment findings suggests a problem with the client's cranial nerves?

A client's extraocular movements are asymmetrical and she complains of diplopia. Explanation: Deficits in cranial nerves III, IV, and VI can manifest as impaired extraocular movements or diplopia. Flashes of light are associated with retinal detachment, while intraocular bleeding and cataracts do not have a neurological etiology.

The review of systems component of the health history is best described as which of the following?

A focus on symptoms related to each of the different body systems Explanation: The review of systems is a systematic method of addressing symptoms, rather than specific diseases, of the major body systems. Because it is an overview, there should not be a large number of questions about each system. Given the focus on identifying symptoms, it is simplistic to describe it as simply a series of head-to-toe questions.

The nurse notes a tophus of the ear of an older adult. Which assessment data is consistent with a tophus?

A hard nodule composed of uric acid crystals Explanation: A tophus is a hard nodule composed of uric acid crystals. A cyst on the ear would present as a fluid-filled sac. Redness and bulging of the eardrum is characteristic of otitis media with effusion. Scarring of the tympanic membrane occurs with repeated ear infections with perforation of the tympanic membrane

A nurse finds crepitus when palpating over a client's maxillary sinuses. Which of the following should the nurse most suspect in this client?

A large amount of exudate in the sinuses Explanation: Frontal or maxillary sinuses are tender to palpation in clients with allergies or acute bacterial rhinosinusitis. If the client has a large amount of exudate, you may feel crepitus upon palpation over the maxillary sinuses. Normal, air-filled sinuses would not demonstrate crepitus. Obstruction of the nostril by a foreign object would prevent sniffing or blowing air through the nostrils, but would not produce crepitus. A perforated septum would also not produce crepitus.

You are assessing visual fields on a client newly admitted for eye surgery. The client's left eye repeatedly does not see your fingers until they have crossed the line of gaze. You would document that the client has what?

A left temporal hemianopsia Explanation: When the client's left eye repeatedly does not see your fingers until they have crossed the line of gaze, a left temporal hemianopsia is present.

The nurse recognizes that an example of subjective data would include:

A pain rating of 7 Explanation: Subjective data include signs and symptoms the client reports. Objective data are data cues the nurse can observe, while subjective data may not be observable to the nurse.

The nurse notes a cyst on the ear of an older adult. Which assessment data is consistent with a cyst?

A sac with a membranous lining filled with fluid Explanation: A cyst on the ear would present as a fluid-filled sac. A tophus is a hard nodule composed of uric acid crystals. Redness and bulging of the eardrum is characteristic of otitis media with effusion. Swelling of the external ear canal with inflammation or infection would be referred to as an edematous ear.

A nursing instructor is discussing mental health assessments with students. In what situations would the instructor tell the students an acute mental health assessment is necessary?

A situation that involves danger of harm to self or others Explanation: An acute mental health assessment includes questions about harm to self or others. Acute situations include a risk for injury that accompanies psychotic states, depression, dementia, or delirium. It is important to ask the safety questions first and leave the presenting problem last. Clients with schizophrenia do not always present with risk for harm to self or others. A client with severe depression is not necessarily at risk of harm to self or others nor is a delirious client.

As a nurse is adjusting a client's hospital bed, the nurse accidently pinches a finger between the bed and the wall. Which of the following components is involved in the transduction of the pain the nurse feels?

A-delta fibers Explanation: Nociceptors are located at the peripheral ends of both myelinated nerve endings of type A fibers and unmyelinated type C fibers, and there are three types that are stimulated by different stimuli: mechanosensitive nociceptors (of A-delta fibers), sensitive to intense mechanical stimulation (e.g., pliers pinching skin); temperature-sensitive (thermosensitive) nociceptors (of A-delta fibers), sensitive to intense heat and cold; and polymodal nociceptors (of C fibers), sensitive to noxious stimuli of a mechanical, thermal, or chemical nature. There are no "K-fibers" or "L-beta fibers."

Where is the temporal artery palpated?

Above the cheek bone near the scalp line Explanation: The nurse palpates the temporal artery in the space above the cheek bone near the scalp line. The temporal artery is not found at midline at the base of the neck, between the mandibular joint and the base of the ear, or just left or right of the spine at the base of the skull.

The nurse is completing a mental health assessment. When the nurse asks the client to interpret a proverb, the nurse is assessing which of the following?

Abstract reasoning Explanation: To test abstract reasoning and comprehension, the nurse might give the client a proverb to interpret. To test attention and concentration, the nurse asks the client, without a pencil or paper, to start with 100 and subtract 7 until reaching 65 or start with 20 and subtract 3. There are four spheres of memory to check: recall, or immediate, memory; short-term memory; recent memory; and long-term, or remote, memory. Insight and judgment are related concepts that involve the ability to examine thoughts, conceptualize facts, solve problems, think abstractly, and possess self-awareness. Insight is the person's awareness of his or her own thoughts and feelings and ability to compare them with the thoughts and feelings of others.

During assessment, the nurse asks a client to explain what the following means: "A penny saved is a penny earned." The nurse is assessing which of the following?

Abstract reasoning Explanation: When assessing abstract reasoning and comprehension, the client is given a proverb such as "a penny saved is a penny earned" and asked to interpret it. Affect is assessed by observing the client's outward expression of emotion. Attention and concentration are assessed by having a client subtract a specific number from a starting number and work backward or asking a client to spell "world" backward.

A woman and her teenager have come to the clinic. The teenager has acne lesions and says that the lesions are not well controlled. The mother asks the nurse what causes acne. What would be the nurse's best response?

Acne is caused by the impedance of sebum secretion onto the skin's surface Explanation: As children approach puberty, the apocrine glands enlarge and become active. At puberty, sebaceous glands increase activity, resulting in large amounts of sebum secreted into the hair follicles of the face, neck, chest, and back. Anything impeding sebum secretion onto the skin's surface may result in the formation of closed comedones and ultimately acne.

A client presents to the health care clinic with reports of two-day history of sore throat pain, ear pressure, fever, and stiff neck. The client states they have taken Tylenol and lozenges without relief. Which nursing diagnosis can be confirmed by this data?

Acute Pain related to sore throat Explanation: The client describes pain on two-day duration which is within the definition for acute pain. The client did not describe or display any major defining characteristics of anxiety such as restlessness, concern about lifestyle changes, sleep disturbances. The stiff neck was not confirmed by objective data collected by the nurse. No evidence exists for the client to have risk for fluid volume deficit.

A client seeks medical attention for pain when touching the area of the frontal sinuses. Which should the nurse consider as the reason for this client's symptom?

Acute bacterial rhinosinusitis Explanation: Frontal sinuses are tender to palpation in clients with acute bacterial rhinosinusitis. This finding would not occur with an eye infection, oropharyngitis, or acute otitis media.

Upon examination of the head and neck of a client, a nurse notes that the submandibular nodes are tender and enlarged. The nurse should assess the client for further findings related to what condition?

Acute infection Explanation: The lymph nodes are enlarged and tender in acute infections. Normally, lymph nodes are not sore or tender and are usually not palpable. Chronic infection causes the nodes to become confluent. In metastatic disease, the nodes enlarge and become fixed in place and are nontender. The lymph node findings may vary in Cushing's disease.

The nurse is assessing a client's lifestyle and habits. At which time should the nurse assess the client for alcohol use?

After assessing for cigarette use Explanation: Questions about alcohol and other drugs follow naturally after questions about cigarettes. Questions about alcohol intake occurs before the review of systems. Alcohol intake is a risk factor that is assessed after vaccinations. Alcohol use is assessed before completing the family history.

Which factor would alert the nurse to the risk of suicide in the client? Select all that apply.

Alcohol abuse Depression Lack of social support Explanation: Risk factors for suicide include irrational thought processes, depression, alcohol abuse, and lack of a spouse or social support.

A nurse notes that a client looks much older than his chronologic age. Which of the following conditions would most likely contribute to this appearance?

Alcoholism Explanation: A client may appear older than actual chronologic age due to a hard life, manual labor, chronic illness, alcoholism, or smoking. Parkinson's disease is associated with stiff, rigid movements. Marfan syndrome is associated with arm span being greater than height and pubis to sole measurement exceeding pubis to crown measurement. Cushing syndrome is associated with central body weight gain with excessive cervical obesity (Buffalos hump).

The student nurse learns that examining the skin can do all of the following except?

Allow early identification of neurologic deficits Explanation: Examination of the skin can reveal signs of systemic diseases, medication side effects, dehydration or overhydration, and physical abuse; allow early identification of potentially cancerous lesions and risk factors for pressure ulcer formation; and identify the need for hygiene and health promotion education.

While completing a history of present illness the nurse asks the client about risk factors. In which way should the nurse use this information?

Analyze as a contributing factor to the current problem Explanation: Risk factors or other pertinent information related to the symptom is frequently relevant, such as risk factors for health problem or a current medication that may have side effects similar to the complaint. Risk factors are not used to determine health teaching, identify a genetic cause, or determine if a family history of the problem exists.

The Kiesselbach plexus is the most common site for what?

Anterior nosebleeds Explanation: The Kiesselbach plexus is the most common site for anterior nosebleeds.

The nurse observes a client's entire body posture to be somewhat stiff, with his shoulders elevated upward toward the ears. The nurse would interpret this to indicate that the client is experiencing which of the following?

Anxiety Explanation: A tense or anxious client may elevate the shoulders toward the ears and hold the entire body stiffly. A restless client would most likely have difficulty sitting, standing still, or keeping body parts still. A slumped body posture may reflect feelings of powerlessness or hopelessness. Confusion is not normally indicated by a particular posture.

A client demonstrates nervousness and fear with a worsening loss of memory. Which nursing diagnosis should the nurse select to help guide this client's care?

Anxiety related to awareness of increasing memory loss Explanation: The client is demonstrating signs of anxiety as evidenced by nervousness and fear. The most appropriate diagnosis would be Anxiety related to awareness of increasing memory loss. There is no evidence that the client has dementia. There is not enough information to determine if the client is disabled or has a hearing loss.

Which of the following statements most accurately describes the maintenance of normal intraocular pressure?

Aqueous humor is continuously circulating through the eye with production equaling drainage. Explanation: Aqueous humor, produced by the ciliary body, maintains intraocular pressure with production equaling drainage. It is not a closed system, and pressure is not adjusted through muscular control of eye volume.

The nurse is inspecting the cornea and lens of an elderly client and notices a white arc around the limbus of the client's eye. The nurse recognizes this condition, common in older adults, as which of the following?

Arcus senilis Explanation: Arcus senilis, a normal condition in older clients, appears as a white arc around the limbus. The condition has no effect on vision. Presbyopia, which is impaired near vision, is caused by decreased accommodation and is a common condition in clients over 45 years of age. Ectropion is when the lower eyelids evert, causing exposure and drying of the conjunctiva. This is a normal finding in the older client. Myopia is impaired far vision.

The nurse notes multiple elevated masses with irregular transient borders that are superficial, raised, and erythematous in a client who complains of an "itching rash." Which question would be most important for the nurse to ask?

Are you allergic to foods, medications, or other substances? Explanation: The lesions most likely appear to be urticaria, which is caused by capillary dilatation in response to an allergic reaction. Asking about anyone else in the family with a similar rash might be appropriate if the lesions were vesicles or pustules. Once the nurse determines the possible cause of the rash, it would be appropriate to gather additional information such as a history of a previous or similar rash and measures to address the itching.

A factory worker has presented to the occupational health nurse with a small wood splinter in his left eye. The nurse has assessed the affected eye and irrigated with warm tap water, but the splinter remains in place. What should the nurse do next?

Arrange for worker to be promptly assessed by an eye specialist. Explanation: The nurse should refer the client to an eye doctor immediately if a foreign body cannot be removed with gentle washing. Optometrists are specialists in primary vision care and do not normally treat eye trauma. Irrigation with hydrogen peroxide or attempted removal using instruments would be contraindicated and potentially dangerous.

A 82 year old female presents with neck pain, decreased strength and sensation of the upper extremities. The nurse identifies that this could be related to what?

Arthritic changes of the cervical spine Explanation: Arthritic changes in cervical spine may present in the older adults as neck pain, decreased strength and sensation of the upper extremities. Bacterial thyroiditis has neck swelling and cranial damage may manifest as headaches or tension of the muscles

An 81-year-old client complains of neck pain and demonstrates decreased range of motion on examination. Which of the following causes should the nurse most suspect in this client?

Arthritis Explanation: Older clients who have arthritis or osteoporosis may experience neck pain and a decreased range of motion. Sudden head and neck pain seen with elevated temperature and neck stiffness may be a sign of meningeal inflammation. Stress and tension may increase neck pain. Neck pain may accompany muscular problems or cervical spinal cord problems.

The nurse is assessing the skin condition and color of an African-American client. Which of the following would the nurse document as an abnormal finding?

Ashen gray skin color Explanation: In dark-skinned individuals, loss of red tones and ashen gray color (suggesting cyanosis) would be considered abnormal. Normally, skin color is evenly distributed; pores may or may not be clearly visible. Color typically ranges from light tan to dark brown or olive in dark-skinned clients.

A mother of a small child calls the clinic and asks to schedule an appointment for ear tube removal. The call is transferred to the nurse. What is the nurse's best action?

Ask the mother how long the tubes have been in place. Explanation: Ear tubes generally fall out spontaneously in 2-5 years after placement, and the membrane most often closes. The client does not need manual removal in the office or operating room unless the child is experiencing problems. Antibiotics are indicated for infection and are not necessary for removal.

Which technique by the nurse demonstrates proper use of the ophthalmoscope?

Asks the client to fix the gaze upon an object and look straight ahead Explanation: After turning on the ophthalmoscope, the nurse should ask the client to gaze straight ahead and slightly upward. Ask the client to remove glasses but keep contact lens in place. The nurse should use the right eye to examine the right eye & left eye to examine the client's left eye. This allows the nurse to get as close as possible to the client's eye. Begin about 10-15 inches from the client at a 15 degree angle. The nurse should keep the ophthalmoscope still & ask the client to look into the light to view the fovea and macula.

A client tells the clinic nurse that she has sought care because she has been experiencing excessive tearing of her eyes. Which assessment should the nurse next perform?

Assess the nasolacrimal sac. Explanation: Excessive tearing is caused by exposure to irritants or obstruction of the lacrimal apparatus. Therefore the nurse should assess the nasolacrimal sac. Inspecting the palpebral conjunctiva would be done if the client complains of pain or a feeling of something in the eye. The client is not exhibiting signs of problems with muscle strength, such as drooping, so performing the eye position test, which assesses eye muscle strength and cranial nerve function, is not necessary. Testing the pupillary reaction to light evaluates pupillary response and function of the oculomotor nerve.

A client is reporting pain and informs the nurse that it has become unbearable. The first thing the nurse should do is what?

Assess the site and intensity of the pain. Explanation: When a client reports pain, the nurse must do an immediate pain assessment. Such an assessment is the first step of the nursing process. The complete pain assessment will cover different characteristics of the pain; however, the very first aspect is to ask about the location and intensity of the pain. Checking for the client's allergies and what medication is ordered will follow after the assessment. The nurse would not call the physician at this point.

A 66-year-old woman has come to the clinic with complaints of increasing fatigue over the last several months. She claims to frequently feel lethargic and listless and states that, "I can never seem to get warm, no matter what the thermostat is set at." How should the nurse proceed with assessment?

Assess the woman for hypothyroidism. Explanation: Fatigue, weakness, and cold sensitivity are symptoms of hypothyroidism. These symptoms are not associated with Cushing's syndrome, hyperthyroidism, or any of the disorders that result in parotid gland enlargement.

Data collection occurs where in the nursing process?

Assessment Explanation: Assessment is the collection of data. Evaluation assesses whether the outcome criteria have been met and revising the plan as necessary. Diagnosis occurs when the data has been analyzed and a professional judgement occurs. Planning is determining outcome criteria and developing a plan.

After teaching a group of students about the phases of the nursing process, the instructor determines that the teaching was successful when the students identify which phase as being foundational to all other phases?

Assessment Explanation: Assessment is the first and most critical phase of the nursing process. If data collection is inadequate or inaccurate, incorrect nursing judgments may be made that adversely affect the remaining phases of the process.

Alexandra, 28 years old, presents to the clinic. She has abdominal pain that she describes as a dull ache, located in the right upper quadrant, and that she rates as a 3 at the least and an 8 at the worst. The pain started a few weeks ago; it lasts for 2 to 3 hours at a time, comes and goes, and seems to be worse a few hours after eating. The client has noticed that the pain starts after eating greasy foods, so she has cut down on this as much as she can. Initially the pain occurred once a week, but now it happens every other day. Nothing makes it better. From this description, which of the attributes of a symptom has been omitted?

Associated symptoms and signs Explanation: The interviewer has not recorded whether nausea, vomiting, fever, chills, weight loss, and so on have accompanied the pain. Associated manifestations are additional symptoms that may accompany the initial chief complaint and that help the examiner to start refining his or her differential diagnosis.

A client arrives complaining of nasal congestion, drainage of a thick, yellow discharge from the nose, difficulty breathing through the nose, headache, and pressure in the forehead. The nurse suspects sinusitis. Which of the following risk factors should the nurse assess for in this client?

Asthma Explanation: This client shows symptoms of sinusitis. Risk factors for sinusitis include a nasal passage abnormality, aspirin sensitivity, cystic fibrosis, chronic obstructive pulmonary disease (COPD), an immune system disorder, hay fever, asthma, and regular exposure to pollutants such as cigarette smoke. The other answers listed—chewing betel nuts, exposure to the sun, and heavy alcohol use—are all risk factors for oropharyngeal cancer, but not for sinusitis.

A client asks a nurse to look at a raised lesion on the skin that has been present for about 5 years. Which is an "ABCDE" characteristic of malignant melanoma?

Asymmetrical shape Explanation: Malignant melanomas are evaluated according to the mnemonic ABCDE: A for asymmetrical, B for irregular borders, C for color variations, D for diameter exceeding 1/8 to 1/4 of an inch, and E for elevated.

A client performs the test for distant visual acuity and scores 20/50. How should the nurse most accurately interpret this finding?

At 20 feet from the chart, the client sees what a person with good vision can see at 50 feet. Explanation: The Snellen chart tests distant visual acuity by seeing how far the client can read the letters standing 20 feet from the chart. The top number is how far the client is from the chart and the bottom number refers to the last line the client can read. A reading of 20/50 means the client sees at 20 feet what a person with normal vision can see at 50 feet. The minus number is the number of letters missed on the last line the client can distinguish.

During the physical examination of a client, a nurse notes that a client's trachea has been pushed toward the right side. The nurse recognizes that the pathophysiological cause for this finding is related to what disease process?

Atelectasis Explanation: Atelectasis can cause the trachea to be pushed to one side from its midline position. Endocarditis is an infection in the muscle of the heart which does not cause the trachea to shift. Bronchitis is an inflammation of the mucous membrane of the bronchial tubes. Tuberculosis is an infection in the lungs. Both bronchitis and tuberculosis are not responsible for the tracheal shift.

A nurse palpates an elderly client's thyroid and detects an enlargement over the right lateral lobe. What action should the nurse take first?

Auscultate with the bell over the lateral lobes Explanation: If a nurse palpates an enlargement of the thyroid, auscultation should be performed with the bell of the stethoscope to assess for the presence of a bruit. A bruit is a soft, swishing sound produced because of an increase in blood flow through the thyroid arteries. The nurse should also ask the client about past history of thyroid problems, the findings must be documented, then the health care provider notified once assessment is complete to obtain further orders.

Which assessment technique should a nurse use to assess for the presence of a bruit in a client with hyperthyroidism?

Auscultation Explanation: A bruit is a soft, blowing, swishing sound auscultated over the thyroid lobes with the bell of the stethoscope that is often heard in hyperthyroidism because of an increase in blood flow through the thyroid arteries. A bruit can be elicited through auscultation in a client with hyperthyroidism. A bruit cannot be elicited through inspection, palpation, and percussion. Inspection can only reveal swelling of the neck and palpation can indicate only the enlarged mass.

The nurse is auscultating a client's blood pressure and identifies the portion of the blood pressure cycle reflecting the break in sounds occurring between the first and second sounds. This is known as which of the following?

Auscultatory gap Explanation: The auscultatory gap refers to the break in or loss of sound during the latter part of phase I (the first appearance of faint, clear, repetitive tapping sounds) and during phase II (muffled or swishing sounds that are softer and longer than phase I sounds). Korotkoff's sounds are the sounds created when measuring blood pressure. Phase V reflects the disappearance of all sounds. Diastolic value refers to the last sound heard before a period of continuous silence and marks the onset of Phase V.

A 72-year-old man comes to the clinic with his daughter for a follow-up visit after a recent hospitalization. He had been admitted to the local hospital for speech problems and weakness in his right arm and leg. On admission his MRI showed a small stroke. The client was in rehabilitation for 1 month following his initial presentation. He is now walking with a walker and has good use of his arm. His daughter complains, however, that everyone is still having trouble communicating with him. The nurse asks the client how he thinks he is doing. Although it is hard to make out his words, the nurse believes the client's answer is "well . . . fine . . . doing . . . okay." His prior medical history involved high blood pressure and coronary artery disease. He is a widower and retired handyman. He has three children who are healthy. He denies tobacco, alcohol, or drug use. He has no other current symptoms. On examination he is in no acute distress but does seem embarrassed when it takes him so long to answer. Blood pressure is 150/90; other vital signs are normal. Other than his weak right arm and leg, physical examination findings are unremarkable. What disorder of speech does he have?

Broca's aphasia Explanation: In Broca's aphasia, clients articulate very slowly and with great effort. Nouns, verbs, and important adjectives are usually present, and only small grammatical words are dropped from speech. Broca's area is on the lateral portion of the frontal lobes.

A nurse cares for a client with a stage II pressure ulcer on the right hip. The nurse anticipates finding what type of appearance to the skin over this area?

Broken with the presence of a blister Explanation: A stage II pressure ulcer results in a superficial skin loss of the epidermis alone or the dermis also. A stage I pressure ulcer is red in color but without skin breakdown. Stage III pressure ulcers involve the epidermis, dermis, and subcutaneous tissue. In stage IV, the muscle, bone, and other supportive tissue may be involved.

The nurse is performing the assessment shown. What is the nurse assessing in this client?

Buccal mucosa Explanation: Using a tongue blade and looking at the inside of the cheek is assessing buccal mucosa. This technique is not used to assess dentition, tongue alignment, or glandular function.

When collecting data on the history of the present illness, it is appropriate to include what?

C-character O- onset L- location D-duration S- severity P- Pattern A- Associated factors

A nurse is assessing the head and neck of an adult client. Which vertebra should the nurse identify as a landmark in order to locate the client's other vertebrae?

C7 Explanation: The vertebra prominens is C7, which can easily be palpated when the neck is flexed. Using C7 as a landmark helps the nurse to locate other vertebrae.

What is a useful tool when assessing a client for alcohol abuse?

CAGE Explanation: The CAGE tool assesses for alcoholism by asking questions about the need to cut down on drinking, feeling annoyed by criticism of drinking, having guilt about drinking, and taking an "eye-opening" drink first thing in the morning. HOPE is used to assess spirituality, SAD PERSONAS is used to assess risk of suicide, and QUIT is not an assessment tool.

The nurse is conducting an assessment of an older adult client who has a diagnosis of chronic heart failure. How can the nurse best assess the effects of the client's stroke volume?

Calculate the difference between the diastolic and systolic pressures. Explanation: The stroke volume is reflected by the pulse pressure, which is the difference between the diastolic and systolic blood pressures. Taking the blood pressure while the client is standing, measuring the strength of the radial pulse, or adding the radial pulse to the systolic blood pressure would not be appropriate to determine stroke volume.

The nurse is caring for a post-operative client with an order for morphine sulfate 2 mg IV push every 4 hours. The client's pain is unrelieved 30 minutes following administration of the morphine sulfate with the pain rating increasing from 7 to 10. Which action should the nurse take?

Call the prescribing physician see about changing the pain medication. Explanation: Untreated or undertreated acute pain may lead to chronic pain syndrome (CRPS). Patients who have had surgery are at increased risk for developing CRPS. The nurse works diligently to find acceptable strategies to address a client's pain, while observing the rights of medication administration. The nurse may need to contact the physician for adjustments in dosing, frequency of dosing, or acquiring an order for another pain medication to obtain optimal pain management for the client. The other three options do not address the client's pain.

Why is it important to collect a thorough and accurate subjective history in regards to a client's nail problems?

Can be caused by an underlying systemic illness Explanation: Diseases or disorders of the nails can be a local problem or they may be a sign of an underlying systemic disease that needs to be assessed. A nurse should be sensitive when interviewing a client with nail problem because they can be damaging to a person's self image. A nurse should ask questions in a nonjudgmental manner if the client has abnormalities of the nails that are due to poor hygiene.

When inspecting the mouth, the nurse focuses on lateral and vertical surfaces of the tongue and its base, because these are regions where:

Cancers often occur. Explanation: It is important to inspect the sides and undersurface of the tongue and the floor of the mouth, because these are areas where cancer most often develops.

The nurse is assessing a client who has been taking antibiotics for an infection for 10 days. The nurse observes whitish curd-like patches in the client's mouth. The nurse should explain to the client that these spots are most likely

Candida albicans infection. Explanation: Whitish, curd-like patches that scrape off over reddened mucosa and bleed easily indicate "thrush" (Candida albicans) infection.

A golden yellow pigment that is heavily keratinized and is found in subcutaneous fat is called what?

Carotene Explanation: Carotene is a golden yellow pigment that exists in subcutaneous fat and in heavily keratinized areas such as the palms and soles.

Which vessel is the nurse assessing if the major artery of the neck is being examined?

Carotid Explanation: The common carotid artery exits the aorta and extends upward in the neck to branch into the internal and external carotid arteries. It is the major artery carrying blood to the brain. The internal jugular veins are located in the neck. The temporal artery is located between the top of the ear and the eye. The radial artery is located at the wrist.

What is the rationale for asking the client whether he or she has noticed any new or changed moles?

Changes in existing moles or the appearance of new moles can indicate melanoma. Explanation: Assessment of moles, both by client and clinician, is important in the early detection of melanomas. Moles are not a relevant finding in cases of psoriasis, vitamin D deficiency, and excess sweat production.

A nurse documents periods of deep breathing followed by periods of apnea. The appropriate term for this type of breathing is

Cheyne-Stokes

A nurse observes the posture of a male client and finds him leaning forward and bracing himself while sitting on the exam table. Which of the following would the nurse most likely suspect?

Chronic obstructive pulmonary disease Explanation: Leaning forward and bracing assists with greater lung expansion and easier breathing in clients with chronic obstructive pulmonary disease. Other deficits or disorders would present with other types of positioning.

A nurse draws a genogram to help organize and illustrate a client's family history. Which shape is a standard format for representing a deceased female relative?

Circle with a cross Explanation: The standard format for representing a deceased female relative in a genogram is using a circle with a cross. A simple circle indicates a living female relative. A simple square indicates a living male relative. A square with a cross indicates a deceased male relative

What will be the nurse's initial role when conducting a health assessment with a client reporting abdominal pain?

Collecting data regarding the nature of the pain Explanation: The nurse's initial role in health assessment is to collect data. Teaching would occur later in the process. Planning care and identifying interventions are parts of the nursing process and not the health assessment.

Assessment of a client who has suffered a recent stroke reveals that he is unresponsive to all stimuli and his eyes remain closed. The nurse documents the client's level of consciousness as which of the following?

Coma Explanation: Coma reflects a client who is unresponsive to all stimuli, with the eyes remaining closed. Obtunded indicates that the client opens eyes to loud voice, responds slowly with confusion, and seems unaware of the environment. With stupor, the client awakens to vigorous shake or painful stimuli but returns to unresponsive sleep. With lethargy, the client opens eyes, answers questions, and falls back to sleep.

This type of assessment includes a health history and physical assessment.

Comprehensive Explanation: A comprehensive assessment includes a detailed health history and a physical assessment. A ongoing assessment occurs after the comprehensive and is a mini-overview. A focus assessment has a problem-oriented focus. An emergency assessment during life-threatening situations.

The thin mucous membrane that lines the inner eyelid and covers the sclera is known as what?

Conjunctiva Explanation: The conjunctiva is a thin mucous membrane that lines the inner eyelid (palpebral conjunctivae) and also covers the sclera (bulbar conjunctivae). The border between the cornea and the sclera is the limbus. The lacrimal apparatus protects and lubricates the cornea and the conjunctiva by producing and draining tears. The eyelid is a loose fold of skin that covers and protects the eye.

A light is pointed at a client's pupil, which then contracts. It is also noted that the other pupil contracts as well, though it is not exposed to bright light. Which of the following terms describes this latter phenomenon?

Consensual reaction Explanation: The constriction of the contralateral pupil is called the consensual reaction. The response of the ipsilateral eye is the direct response. The dilation of the pupil when focusing on a close object is the near reaction. Accommodation is the changing of the shape of the lens to sharply focus on an object.

A nurse shines a light into one eye during ocular exam and the pupil of the other constricts. The nurse interprets this as which of the following?

Consensual response Explanation: When a light is shone in one eye, that eye will constrict and the opposite (consensual) eye will also constrict. Shining a light on one eye with the resulting constriction of that eye demonstrates the direct reflex. The optic chiasm is the point where the optic nerves from each eyeball cross. Accommodation occurs when the client moves the focus of vision from a distant point to a near object, causing the pupils to constrict.

When palpating the lymph nodes of the neck, the nurse assesses for which of the following characteristics?

Consistency, delineation, mobility, tenderness Explanation: Parameters of lymph node assessment include size, shape, delimitation, mobility, consistency, and tenderness.

A nurse is inspecting the ears of an Asian client and observes that her earlobes appear soldered, or tightly attached to adjacent skin with no apparent lobe. Which of the following should the nurse do next?

Continue with the examination Explanation: Earlobes may be free, attached, or soldered (tightly attached to adjacent skin with no apparent lobe). Most African Americans and Caucasians have free lobes, whereas most Asians have attached or soldered lobes, although any type is possible in all cultural groups. Thus, this finding is normal and does not need to be reported to the physician, followed up on with a question to the client about an ear injury, or recorded and followed up on at a later visit.

A client has been diagnosed with astigmatism. The nurse should be prepared to teach the client about which treatment for this condition?

Corrective lenses Explanation: Astigmatism is corrected with a cylindrical lens that has more focusing power in one access than the other. These corrective lenses can and should be worn while driving at night. Eye drops and surgery are not usual treatments for this condition.

During the introduction phase of the interview, the client begins to talk nonstop about health problems, family issues, and fears related to illness. What can the nurse do to control the interview process?

Courteously interrupt the client to clarify some information. Explanation: The nurse should only interrupt the client when necessary in a courteous manner. The purpose of the interruption should be to clarify some information that the client provided while talking. Glancing at the clock on the wall and tapping the pen on the paper are signs of impatience and should not be done when working with a talkative client. Contacting security is not necessary when interviewing a talkative client.

A nurse is working with a client who has an impaired ability to move the tongue. He explains that he was in an automobile accident many years ago and suffered nerve damage that resulted in this condition. Which nerve should the nurse suspect was damaged in this client?

Cranial nerve XII (hypoglossal) Explanation: Decreased tongue strength may occur with a defect of the twelfth cranial nerve—hypoglossal—or with a shortened frenulum that limits motion. Receptors of cranial nerve I (olfactory) are located in the nose. These receptors are related to the sense of smell. A loss of taste discrimination occurs with a defect of cranial nerve VII (facial). The palate fails to rise and the uvula deviates to the side with cranial nerve X (vagus) paralysis.

Which factor influences the nurse's ability to individualize the health assessment? Select all that apply.

Cultural factors Sensory deficits Age of individual Explanation: Factors that influence the nurse's ability to individualize the health assessment include the age of the individual, sensory deficits (such as decreased vision, hearing, and cultural beliefs and practices. Family history and lifestyle behaviors do not affect the nurse's ability to individualize the health assessment.

A nurse observes the presence of hirsutism on a female client. The nurse should perform further assessment on this client for findings associated with which disease process?

Cushing's disease Explanation: Hirsutism, or facial hair on females, is a characteristic of Cushing's disease and results from an imbalance of adrenal hormones. Iron deficiency anemia is associated with spoon-shaped nails but not with excessive hair. Carcinoma of the skin causes lesions but not facial hair. Lupus erythematosus causes patchy hair loss but does not cause excessive facial hair.

When depression goes undiagnosed, what consequences occur eight times more frequently than in the general population?

Death Explanation: Failure to diagnose depression can have fatal consequences-suicide rates among clients with major depression are eight times higher than in the general population.

The nurse is admitting a client to surgical day care and is assessing the client's vital signs. When obtaining the client's oral temperature, where should the nurse insert the thermometer?

Deep in the posterior sublingual pocket Explanation: When taking an oral temperature, the nurse places the thermometer under the client's tongue to the right or left of the frenulum deep in the posterior sublingual pocket. The thermometer is not placed between the cheek and tongue, at the gingival level, or just below the tongue after the teeth.

An emergency department nurse has utilized the Confusion Assessment Method (CAM) in the assessment of a 79-year-old client with a new onset of urinary incontinence. This assessment tool will allow the nurse to confirm the presence of what health problem?

Delirium Explanation: The CAM assesses for delirium; it does not assess for dementia, schizophrenia, or psychosis.

A nurse is working with a 13-year-old boy who complains that he has begun to sweat a lot more than he used to. He asks the nurse where sweat comes from. The nurse knows that sweat glands are located in which layer of skin?

Dermis Explanation: The dermis is a well-vascularized, connective tissue layer containing collagen and elastic fibers, nerve endings, and lymph vessels. It is also the origin of sebaceous glands, sweat glands, and hair follicles. The epidermis, the outer layer of skin, is composed of four distinct layers: the stratum corneum, stratum lucidum, stratum granulosum, and stratum germinativum. The outermost layer consists of dead, keratinized cells that render the skin waterproof.

Which layer of the skin contains blood vessels, nerves, sebaceous glands, lymphatic vessels, hair follicles, and sweat glands?

Dermis Explanation: The second layer, the dermis, functions as support for the epidermis. The dermis contains blood vessels, nerves, sebaceous glands, lymphatic vessels, hair follicles, and sweat glands, which support the nutritional needs of the epidermis and provide support for its protective function. the top layer of the skin is the dermis layer outermost skin layer, and serves as the body's first line of defense against pathogens, chemical irritants, and moisture loss. The subcutaneous layer provides insulation, storage of caloric reserves, and cushioning against external forces. Composed mainly of fat and loose connective tissue, it also contributes to the skin's mobility. The connective layer is a distracter to the question.

The nurse documents that a client completed a 4-year college program and speaks English. How will this information be used?

Determine health literacy Explanation: Knowledge of the client's education level and primary language will help assess the client's health literacy level which is the degree to which individuals have the capacity to obtain, process, and understand basic health information and services needed to make appropriate health decisions. Education level and primary language are not used to validate the client's age, understand the choice of occupation, or to analyze the client's lifestyle patterns.

A nurse examines a client's retina during the ophthalmic examination and notices light-colored spots on the retinal background. The nurse should ask the client about a history of what disease process?

Diabetes Explanation: Exudates appear as light-colored spots on the retinal background and occur in individuals with diabetes or hypertension. Anemia, renal insufficiency, and retinal detachment do not cause this appearance on the retina.

The nurse is collecting information regarding the individual's medications. Which nursing activities does the nurse perform during this component of the health history? Select all that apply.

Differentiate among adverse versus allergic responses to medications Validate names and dosages of medications currently being taken Assess the individual's understanding of each medication's purpose Explanation: Nursing activities include validating the names and dosages of medications currently being taken, assessing the individual's understanding of each medication's purpose, and differentiating among adverse versus allergic responses to medications. The nurse does not document any adverse effect as an allergic response to protect the client and note the pharmaceutical company making the client's medications.

A nurse on an oncology unit enters a client's room to auscultate bowel sounds. What should the nurse do before auscultating?

Disinfect the stethoscope before touching the client Explanation: The nurse makes sure to disinfect the stethoscope between clients to avoid the spread of pathogens. Disinfecting the stethoscope after touching the client does not answer the question being asked. Placing the stethoscope directly on the client's skin does not answer the question being asked. Nothing noted in the question would require the nurse to wear a personal protection gown.

The terms "generalized," "exposed surfaces," "upper arm," and "skin folds" are used to describe which major characteristic of skin lesions?

Distribution Explanation: The given terms denote anatomic location, or distribution, of skin lesions over the body.

Which clients are most at risk for depressive symptoms? (Select all that apply.)

Divorced clients Females Chronically ill clients Explanation: Watch carefully for depressive symptoms, especially in clients who are young, female, single, divorced or separated, seriously or chronically ill, or bereaved. Those with a prior history or family history of depression are also at risk.

A nurse has performed a head and neck assessment of an adult client and noted that the thyroid gland is not palpable. What is the nurse's most appropriate action?

Document this as an expected assessment finding Explanation: It is not unusual for the thyroid lobes to be non-palpable using the posterior approach.

The nurse would use what part of the hand when assessing temperature during palpation?

Dorsal surface Explanation: The dorsal surface is used for temperature. The finger pads are used for fine discrimination such as pulses, texture and size. The ulnar or palmar surface is used for vibrations, thrills and fremitus.

The nurse is beginning the examination of the skin of a 25-year-old teacher. She previously visited the office for evaluation of fatigue, weight gain, and hair loss. The previous clinician had a strong suspicion that the client has hypothyroidism. What is the expected moisture and texture of the skin of a client with hypothyroidism?

Dry and rough Explanation: A client with hypothyroidism is expected to have dry and rough skin. This is a good example of how the skin can give clues to systemic diseases.

During assessment of the oral cavity, the nurse examines the salivary glands. Which area of the mouth should the nurse assess to inspect for the Wharton's ducts?

Either side of the frenulum on the floor of the mouth Explanation: The nurse should inspect the Wharton's ducts on either side of the frenulum on the floor of the mouth. Stenson's ducts, not Wharton's ducts, are visible on the buccal mucosa across from the second upper molars. The right side of the frenulum at the base of the gums and on the posterior aspect of the tongue bilaterally are not appropriate sites to inspect for salivary ducts. Reference:

An adult client is brought to the ED after falling 12 feet from a ladder. The client has an obvious deformity to his left lower leg. What kind of assessment is the nurse going to perform?

Emergency Explanation: An emergency assessment occurs when the client's condition is unstable. A focused assessment covers one subject, usually the current illness. A comprehensive assessment covers every system in the body, including a past history and a family history. A head-to-toe assessment is a complete physical assessment of the body.

A client arrives at the Emergency Department reporting shortness of breath. She is cyanotic with bilateral wheezing. The client begins to gasp for air and cannot speak. The nurse begins to gather information so that interventions can resolve the immediate breathing problem. Her assessment and interventions are concurrent. The nurse is performing what type of health history?

Emergency Explanation: The nurse is performing an emergency health history, the purpose of which is to collect the most important information and defer obtaining details until the client is stable. The focused health history involves questions that relate to the current situation. The comprehensive health history takes place during an annual physical examination. There is not a primary health history for clients.

The nurse is discharging an adult client who received 18 staples for a head laceration received while mountain biking. What can the nurse focus on while doing discharge teaching?

Encourage the use of safety equipment Explanation: Nurses encourage use of appropriate safety equipment to reduce risk of head or neck trauma. There is no identified need to encourage proper nutrition to promote healing in this client. There is no identified need to teach proper posture, bending, and lifting with this client. Encouraging the client to take a safety course is not the primary focus of discharge teaching.

A young adult client has come to the clinic for her scheduled Pap (Papanicolaou) test and pelvic examination. The nurse would implement which action to help reduce the client's anxiety during the physical exam?

Ensuring client's privacy by providing an examination gown Explanation: The client is usually concerned about unnecessary body exposure. Explanation and reassurance that the nurse will protect the client's privacy decreases this anxiety. Providing a comfortable, warm room temperature is appropriate to prevent chilling, but is usually less important to the client than privacy. Arranging exam equipment on a bedside tray table if within the view of the client may add to the client's anxiety. However, arranging the exam equipment would facilitate organization. Explaining why standard precautions are being used may help alleviate some anxiety, but the client probably will not understand what standard precautions are.

A nurse is examining a child who is suspected of having bronchitis and is preparing to auscultate his chest with a stethoscope. Which of the following actions would demonstrate the correct technique for this procedure?

Ensuring that contact with the skin is maintained Explanation: While using a stethoscope to listen to air movement through the respiratory tract, the nurse should avoid listening through clothing, as it may obscure or alter the sound. However, too much pressure should not be applied when using the bell, as it would cause it to work like a diaphragm. The diaphragm is used to listen to high-pitched sounds, whereas the bell is used to listen to low-pitched sounds.

When observing a client diagnosed with mania, the nurse observes his mood to be elated. Another term for this type of mood includes which of the following?

Euphoric Explanation: Terms used to describe mood include euthymic (normal), euphoric (elated), labile (changeable), and dysphoric (depressed, disquieted, restless).

A nurse has been asked to complete a mental status examination of a psychiatric-mental health client. Which of the following is included in this assessment?

Evaluation of insight and judgment Explanation: The mental status examination is a central aspect of the psychiatric assessment process that assesses current cognitive and affective functioning through data collection on appearance, behavior, level of consciousness, speech, thought content and processes, cognitive ability, mood and affect, insight, and judgment. This assessment relies almost exclusively on observation rather than inquiry and is expected to change during treatment.

The nurse notes a large keloid on the pierced ear of an adolescent. The client asks what caused this finding. Which of the following would the nurse incorporate into the response as the most likely cause?

Excessive collagen formation Explanation: Keloids are caused by excessive collagen formation during the healing process, not from continuous trauma, decreased subcutaneous tissue, or inadequate circulation.

A client complains of excessive tearing of the eyes. Which assessment would the nurse do next?

Excessive tearing is caused by exposure to irritants or obstruction of the lacrimal apparatus. Therefore the nurse should assess the nasolacrimal sac. Inspecting the palpebral conjunctiva would be done if the client complains of pain or a feeling of something in the eye. The client is not exhibiting signs of problems with muscle strength such as drooping, so performing the eye position test, which assesses eye muscle strength and cranial nerve function, is not necessary. Testing the pupillary reaction to light evaluates pupillary response and function of the oculomotor nerve.

What is a characteristic symptom of Graves hyperthyroidism?

Exophthalmos Explanation: In exophthalmos the eyeball protrudes forward. When bilateral, it suggests the infiltrative ophthalmopathy of Graves hyperthyroidism.

A 29-year-old physical therapist presents for evaluation of an eyelid problem. On observation, the right eyeball appears to be protruding forward. Based on this description, what is the most likely diagnosis?

Exophthalmos Explanation: In exophthalmos, the eyeball protrudes forward. If it is bilateral, it suggests the presence of Graves' disease, although unilateral exophthalmos could still be caused by Graves' disease. Alternative causes include a tumor and inflammation in the orbit.

When performing the cover test, a nurse notices that the client's left eye turns outward. How should the nurse document this finding in the client's record?

Exotropia Explanation: With the cover test, the eyes of the client should remain fixed straight ahead. If the covered eye moves when uncovered to reestablish focus, it is abnormal. If the eye turns outward it is called exotropia. If the uncovered eye turns inward, it is called esotropia. Strabismus is constant malalignment of the eyes. Presbyopia is impaired near vision.

A female client is told that she needs a pelvic exam and Papanicolaou (Pap) smear. She says "Absolutely not! There's no way I'll let you do that to me!" Which response by the nurse would be most appropriate?

Explain the importance of the pelvic exam and Pap smear, but respect the client's wishes and omit the exam. Explanation: The nurse should explain to the client the importance of the examination and the risk of missing important information if any part is omitted. However, whether or not to have the examination is the client's decision and must be respected.

The nurse prepares an educational program for the families of clients recovering from burns. On the diagram provided, select the area where fat cells, blood vessels, and nerves are located.

Explanation: Beneath the dermis lies the subcutaneous tissue, a loose connective tissue containing fat cells, blood vessels, nerves, and the remaining portions of sweat glands and hair follicles.

A nurse is performing percussion on a client's back to assess the lungs, and hears a loud, low-pitched, hollow sound, indicating normal lungs. Which of the following describes this finding?

Explanation: Resonance is a loud, low-pitched, hollow sound normally percussed over an area that is part air and part solid, which is expected over normal lung fields. Hyper-resonance is a very loud, low-pitched sound that is normally heard in lungs with a lot of air such as in emphysema. Tympany is a very loud, high-pitched, drum-like sound that is heard over an air-filled structure, such as the stomach. Dullness is a medium-pitched, thud-like sound that is percussed over solid tissue such as

The nurse prepares to assess the anterior triangle of a client's neck. Where should the nurse palpate this area on the diagram?

Explanation: The anterior triangle is located in the area below the mandible, lateral to the sternocleidomastoid muscle and medial to the midline of the neck.

The nurse is preparing to examine a client's skin. What would the nurse do next?

Expose only the body part that is being examined. Explanation: When preparing to examine a client's skin, the nurse would expose only the body part to be examined to ensure privacy. The room should be at a comfortable temperature, one that is not too warm or too cool. Gloves are needed when palpating any lesions. The client needs to remove all clothing and jewelry and put on an examination gown.

A nurse is conducting a mental status assessment of a 70-year-old male client who is being treated for depression. What would the nurse consider when assessing the client's facial expression and eye contact?

Eye contact is strongly influenced by cultural norms. Explanation: Eye contact and facial expressions, such as smiling, differ widely between cultures. Reduced eye contact is not an age-related physiological change. Informing the client that his facial expression is being assessed will likely confound the assessment results. Mental illness does not preclude assessment of eye contact and facial expression.

Which of the following would a nurse expect to assess in a client with esotropia?

Eye turning inward Explanation: Esotropia is a term used to describe eyes that turn inward. Exotropia refers to an outward turning of the eyes. Strabismus refers to a constant malalignment of the eyes. Nystagmus refers to oscillating or shaking movement of the eye.

During a health history, a client reports drinking bloody Mary's several mornings a week before going to work. In which part of the CAGE questionnaire should the nurse document this information?

Eye-openers Explanation: The client drinking alcohol in the morning would be applicable to the area on eye-openers specifically the question "Have you ever taken a drink first thing in the morning (Eye-opener) to steady your nerves or get rid of a hangover? C=cutting down (have you felt the need to cut down?) A=Annoyed by others criticisms G=guilty feelings about drinking E=Eye openers-do you feel the need to drink in the morning?

The nurse suspects that a client is experiencing pain despite denying pain upon assessment. What did the nurse observe to make this clinical determination? Select all that apply.

Facial grimacing Lack of eye contact Heart rate 15 beats higher than usual Respiratory rate shallow and irregular

The frontal sinuses are the only ones readily accessible to clinical examination.

False

When assessing pulses, the nurse would use which part of the hand for palpation?

Finger pads Explanation: The finger pads are used for fine discrimination such as pulses, texture and size. The ulnar or palmar surface is used for vibrations, thrills and fremitus. The dorsal surface is used for temperature.

A nurse receives report from the shift nurse that a client has new onset of peripheral cyanosis. Where should the nurse focus the assessment of the skin to detect the presence of this condition?

Fingers and toes Explanation: Peripheral cyanosis is usually a local problem with manifestations of cyanosis, a blue-tinged color to the skin, caused by problems resulting in vasoconstriction. Changes in color around the mouth are called circumoral. Bluish tints to the chest and abdomen cyanosis is called central cyanosis.

A 29-year-old woman comes to the office. During history taking, the nurse notices that the client is speaking very quickly and jumping from topic to topic so rapidly that it is difficult to follow her. The nurse can find some connections between ideas, but it is difficult. Which word best describes this thought process?

Flight of ideas Explanation: This represents flight of ideas, because the ideas are connected in some logical way. Derailment, or loosening of associations, has more disconnection within clauses. Circumstantiality is characterized by the client speaking "around" the subject and using excessive detail, though thoughts are meaningfully connected. Incoherence lacks meaningful connection and often has odd grammar or word use.

A client with hypertension seeks medical attention for a new onset of a nosebleed. Which type of assessment should the nurse complete with this client?

Focused Explanation: A focused or problem-oriented assessment focuses on the client's current problem. The client's symptoms, age, and this history will determine the extent of the physical examination to perform. An emergent or emergency assessment focuses on a specific problem that may be life-threatening. This type of assessment focuses on circulation, airway, and breathing (CAB) when cardiac arrest is suspected. A follow-up history is a form of a focused assessment. The client is returning to have a problem evaluated after treatment. Data is gathered to evaluate if the

A nurse assesses the parallel alignment of a client's eyes by testing the corneal light reflex. Where should the nurse shine the penlight to obtain an accurate result?

Focused on the bridge of the nose Explanation: When testing the corneal reflex, the nurse should shine the light toward the bridge of the nose. At the same time, the client is instructed to stare straight ahead. This facilitates a parallel image on the cornea. The eye response upon shining the light toward the eye may interfere with the assessment. The light should not be shined toward the forehead or on an object on the wall.

A client with a foot wound returns to the outpatient wound clinic for a weekly appointment and treatment. Which type of assessment should the nurse complete with this client?

Follow-up Explanation: A follow-up history is a form of a focused assessment. The client is returning to have a problem evaluated after treatment. Data is gathered to evaluate if the treatment plan was successful. A focused or problem-oriented assessment focuses on the client's current problem. The client's symptoms, age, and this history will determine the extent of the physical examination to perform. A comprehensive assessment is completed when admitting a client to a facility.

When the nurse questions a client about sitting, rising from a chair, standing for periods, climbing stairs, shopping, driving, and participating in sports, what is he or she assessing?

Function Explanation: Pain can affect the ability to perform common movements and tasks. Assess the effects of pain on the client's functional ability by questioning the client about sitting, rising from a chair, standing for a while, climbing stairs, shopping, driving, and participating in sports. Pain is dynamic and increases with activity.

A client reports a weight loss and fatigue during the review of systems. In which area should the nurse document this information?

General Explanation: Information to document under the general area includes usual weight, recent weight change, any clothes that fit more tightly or loosely than before, weakness, fatigue, or fever. Information about weight and fatigue is not documented under the gastrointestinal system. Appetite and rest and sleep are not areas within the review of systems.

A client on a medical-surgical unit reports pain of 10 on a scale of 0 to 10 and wants more pain medication. The nurse does not think the pain is as bad as the client says. The physician left orders for prn morphine for breakthrough pain. What is the priority nursing action?

Give the prn morphine Explanation: Pain is what the client says it is, and it exists whenever the client says it does. It would not be appropriate to hold the medication for 30 minutes, call the physician to check the order, or just document the client's pain.

A nurse is preparing to perform intubation on a client. Which pieces of equipment are needed to prevent the transmission of infectious agents during this procedure? Select all that apply.

Gloves Gown Face shield Explanation: The specific personal protective equipment needed to prevent the transmission of infectious agents varies depending on the procedure to be performed. For example, performing venipuncture requires only gloves, but intubation requires gloves, gown, and face shield, mask, or goggles. A nasopharyngeal airway may be needed for intubation, but its purpose is not to prevent transmission of infectious agents. A stethoscope would not be needed for this procedure.

What included in personal protective equipment? Select all that apply.

Gloves Gown Mouth, nose, eye protection Explanation: Personal protective equipment (PPE) includes gloves, gown, mouth, nose and eye protection. Special linen and cleaning processes are not part of PPE.

A nurse will be performing a complete physical examination of a man who has emphysema with a chronic productive cough, including an assessment of his oral cavity. Which pieces of personal protective equipment should the nurse wear?

Gloves, mask, protective eye goggles, gown Explanation: Because this client has emphysema with a chronic productive cough, it is likely that the nurse will not only come into direct contact with the client's sputum or mucus (a body fluid) during examination of his oral cavity, which requires the use of gloves, but also that sputum will be sprayed on the nurse's face and body, which requires the use of a mask, protective eye goggles, and a gown.

On palpation, the nurse notes that a client's thyroid gland is diffusely enlarged. Which of the following health problems is associated with this finding?

Graves' disease Explanation: Graves' disease is associated with a diffusely enlarged thyroid. This finding is not normally consistent with neoplasm, hypothyroidism, or nephritic syndrome.

The nurse has made a nursing diagnosis of self-esteem disturbance. Which assessment data supports the nursing diagnosis?

Guilt and negative comments about self Explanation: Guilt and negative comments about self are indicative of self-esteem disturbance. Impaired social interaction is characterized by feeling ill-at ease during social situations. Expressions of hopelessness and loneliness are indicators of risk for suicide. A nursing diagnosis of sensory-perceptual alterations is characterized by poor concentration and hallucinations.

A 22-year-old man is brought to the office by his father. The client was diagnosed with schizophrenia 6 months ago and has been taking medication since. The father states that his son's dose isn't high enough and needs to be increased. He states that his son has been hearing things that don't exist. The nurse asks the young man what is going on. He says that his father is just jealous because his sister only talks to him. His father turns to him and says, "Son, you know your sister died 2 years ago!" His son replies "Well, she still talks to me in my head all the time!" Which best describes this client's abnormality of perception?

Hallucination Explanation: A hallucination is a subjective sensory perception without real external stimuli. The client can hear, see, smell, taste, or feel something that does not exist in reality. In this case, his sister has passed away and cannot be speaking to him, although in his mind he can hear her. This is an example of an auditory hallucination, but hallucinations can occur with any of the five senses.

When caring for clients in any health care environment, what is the most important technique for preventing infection?

Hand hygiene Explanation: The single most important action to prevent infection is hand hygiene. Sterile technique is important in preventing infection in invasive procedures, but the questions does not address the specific situation of invasive procedures, only infection in general. Standard precautions and the use of gloves prevent a healthcare provider from being exposed to blood and body fluid while caring for a client. This is important in the infection cycle, but is not the most important technique.

The nurse is preparing for a physical examination of a client. What should the nurse do first?

Hand hygiene Explanation: The nurse should perform hand hygiene before beginning the physical assessment. This includes prior to gathering equipment. Auscultation and palpitation should not occur until after hand hygiene has been performed.

The nurse is preparing to perform a physical examination of a client who is an Orthodox Jew. Which of the following accommodations should the nurse be prepared to make for this client, based on his religious beliefs?

Have a nurse who is the same sex as the client examine him Explanation: Clients from conservative religious groups (e.g., Orthodox Jews or Muslims) may require that the nurse be the same sex as the client. The client must still undress and put on an examination gown. It is not likely that the client will want to pray before the examination, and it is not necessary to avoid asking questions regarding his lifestyle.

A client complains of pain in several areas of the body. How should the nurse assess this client's pain?

Have the client rate each location separately. Explanation: When assessing pain location, ask the client to point to the painful area. If more than one area is painful, have the client rate each one separately, and note which area is the most painful. Marking each site is not necessary practice for assessing pain. Pain is a subjective sensation for the client. Radiating pain is notable, because such radiation may affect treatment choices.

A nurse needs to assess a client who is experiencing chronic headache to determine how it is affecting her activities of daily living. Which of the following interventions should the nurse implement?

Headache Impact Test Explanation: The Headache Impact Test may be used to assess the impact of headache on a client's activities of daily living. A mnemonic assessment tool is used to assess for the character, onset, location, duration, severity, pattern, and associated factors of pain. It does not assess for the effect of pain on the client's activities of daily living. Auscultation is use of a stethoscope to assess the client's blood pressure, heart sounds, or respiration. The family health history portion of the interview is used to assess for health conditions of family members that might help shed light on the client's chief complaint.

When talking to a client before starting the physical exam, the nurse notes that the client consistently tilts her head to one side. What would the nurse examine first?

Hearing acuity Explanation: A head tilted to one side may indicate unilateral vision or hearing deficiency, which should be ruled out before proceeding with the examination. The nurse would not need to evaluate the thyroid gland, mental status, or lymph nodes based on this finding.

A nurse performs an admission assessment on a client admitted with chest pain. The nurse knows that using the bell of the stethoscope is appropriate to auscultate for which type of sounds?

Heart murmur Explanation: The bell of the stethoscope is used to listen for low pitched sounds such as abnormal heart sounds or bruits. The diaphragm is used to listen for high pitched sounds such as normal heart, lung, & bowel sounds.

The nurse is completing the general survey of a client and determines that the client's temperature is 102°F. Which of the following would the nurse also expect to find?

Heart rate greater than 100 bpm Explanation: The client would most likely exhibit tachycardia due to the increase in heart rate that occurs in response to increased body temperature. The pulse rate increases in an attempt to increase body metabolism and decrease temperature. Weak, thready pulse would be considered abnormal but not necessarily associated with a temperature of 102F. A respiratory rate between 12 and 20 breaths per minute would be normal. Blood pressure of 10 mm Hg greater than normal is not associated with fever.

The eustachian tube is a passage between the middle ear and the nasopharynx. What is the function of the eustachian tube?

Helps to regulate pressure in the middle ear Explanation: The eustachian tube, a conduit that connects the middle ear to the nasopharynx, allows for pressure regulation of the middle ear. The other options do not accurately describe the function of the eustachian tube.

A client presents with a cluster of upper airway complaints that include rhinorrhea. Which area of assessment would yield the most pertinent information to the etiology of rhinorrhea?

History of allergies Explanation: Rhinorrhea (thin, watery, clear nasal drainage) may indicate chronic allergy, which is the primary area for assessment and will yield the most pertinent information. Immunizations are unlikely to relate directly to this sign. Nosebleeds may be seen with overuse of nasal sprays, excessively dry mucosa, hypertension, leukemia, and other blood disorders. Tonsillar enlargement may be associated with tonsillitis or other infectious processes.

A nurse is assessing the effect of a client's chronic back pain on his affective dimension. Which question should the nurse ask for this assessment?

How does the pain influence your overall mood? Explanation: The question regarding the influence of the pain on mood would address the client's affective dimension, which includes feelings and emotions that result from the pain. The question regarding medical conditions would help assess the client's physical dimension. The question regarding the location of the pain would address the client's sensory dimension. The question regarding the client's education would address his cognitive dimension.

When providing client teaching about the ears, what should the nurse be sure to include?

How the client cleans the ears Explanation: It is important to address how the client cleans the ears. Many people associate cerumen in the ear canal with lack of hygiene and therefore clean their ears routinely. Often, clients think that cotton-tipped applicators are for this purpose. This self-care behavior is unsafe, placing clients at risk for cerumen impaction. Nurses should reinforce proper cleaning techniques. Since cleaning with cotton-tipped applicators is not correct, the nurse would not teach the client how to use the applicators to clear the ears. The nurse would not teach the client about basic anatomy and physiology of the ears. The option of potential infection from self-cleaning of ears is not correct.

A staff educator from the hospital is providing an event for the hospital staff. The educator is talking about health promotion activities for people with diseases of the nose, mouth, throat, and sinuses. What would the educator include in the presentation?

How to reduce periodontal disease Explanation: Major risk reduction and health promotion goals in assessment of the nose, sinuses, mouth, and throat are related to various issues, including tobacco use, obstructive sleep apnea, oral health, and cancer. Health goals include reducing periodontal disease.

A client comes to the Emergency Department with bruises on her upper and lower body and appears to be withdrawn. The injuries do not appear consistent with the explanations for them. The client's boyfriend refuses to leave the examination room and is overly protective of her. The nurse suspects:

Human violence Explanation: The indications should raise the nurse's suspicions of abuse of the client by the boyfriend. Commonly, abusers are overly protective in the presence of others and will not leave the examination room. Hypertension, inability to perform ADLs, and the eating disorder anorexia nervosa are not indicated in this scenario of bruising and withdrawal.

A nurse is performing a head and neck assessment of a client who is newly admitted to the hospital unit. When preparing to assess the client's thyroid gland, what landmarks should the nurse first identify? Select all that apply.

Hyoid bone Cricoid cartilage Explanation: Thyroid assessment begins with the identification of relevant landmarks, including the thyroid cartilage, the hyoid bone, and the cricoid cartilage. The sternocleidomastoid muscle, esophagus, and carotid arteries are not landmarked.

What structure is found midline in the tracheal area just beneath the mandible?

Hyoid bone Explanation: Important landmarks for the head and neck region are in the tracheal area. The usually palpable U-shaped hyoid bone is located midline just beneath the mandible. The large thyroid cartilage consists of two flat, plate-like structures joined together at an angle and with a small, sometimes palpable notch at the superior edge. Usually more prominent in males, the thyroid cartilage is also called the "Adam's apple." The palpable cricoid cartilage is a ringed structure just inferior to the thyroid cartilage.

The nurse explains to the client that smoking has what effect on the body? Select all that apply.

Hypertension Vasoconstriction Peripheral vascular disease Explanation: Smoking can cause vasoconstriction, hypertension and peripheral vascular disease, not vasodilation and hypotension.

A client presents to the emergency department after being hit in the head with a baseball bat during a game. The nurse should assess for which condition?

Hyphema Explanation: Hyphema is blood in the anterior chamber of the eye, usually caused by blunt trauma. Blepharitis is inflammation of the margin of the eyelid. Chalazion is a cyst in the eyelid. Iris nevus is a rare condition affecting one eye. The latter 3 conditions are not commonly attributed to blunt force trauma to the head as hyphema is.

A client presents to the health care clinic with reports of new onset of generalized hair loss for the past two months. The client denies the use of any new shampoos, or other hair care products; no new medications. The nurse should ask the client questions related to the onset of which disease process?

Hypothyroidism Explanation: Generalized hair loss can be a finding in hypothyroidism. Diabetes is a problem with glucose regulation. Crohn's disease is an inflammatory process in the large intestines. Liver disease results in many problems with fluid regulation, metabolism of drugs, and storage of glucose.

The nurse notes that a client's capillary refill is 5 seconds. What should this finding indicate to the nurse?

Hypoxia Explanation: When the capillary refill is greater than 2 seconds, a respiratory or cardiovascular disease should be considered as causing hypoxia. This finding does not indicate an infection or a vitamin C deficiency. This is not a normal finding.

In the closing phase of the interview process, the nurse analyzes the data collected for what priority reason?

Identifying the primary problems or patterns of concern Explanation: The nurse prioritizes, collects, and analyzes subjective and objective data and summarizes and states the two to three most important patterns or problems might be. The nurse's priority is not use the data gathered in the client interview as a baseline for interviewing the family or for communicating to the physician or other staff members.

During the health interview, a client reports an occasional blockage in the upper portion of the nasal passage. The nurse understands the most pronounced effect this would have on the client would be what?

Impaired sense of smell Explanation: Receptors for cranial nerve I (olfactory) are located in the upper part of the nasal cavity and septum. Blockage would decrease the ability to smell. A decreased ability to taste would be associated with an upper respiratory infection or lesion of the facial nerve. Difficulty hearing or occasional dizziness is associated with ear and vestibular problems.

A nurse caring for a client admitted 2 days ago following a cerebral vascular accident. The nurse notes that the client is frequently coughing, has food falling from the mouth while eating, and frequently chokes. What would be the most pertinent nursing diagnosis for this client?

Impaired swallowing Explanation: Impaired swallowing is associated with problems in oral, pharyngeal, or esophageal structure or function. Related findings include delayed swallowing; gurgly voice; frequent coughing, choking, or gagging; inability to clear oral cavity; and food falling from the mouth. The scenario described does not mention impaired dentition, alteration in mobility, or altered cerebral perfusion.

The client is having a Weber test. During a Weber test, where should the tuning fork be placed?

In the midline of the client's skull or in the center of the forehead. Explanation: The Weber test is performed by striking the tuning fork and placing its stem in the midline of the client's skull or in the center of the forehead. In the Rinne test, the tuning fork is struck and placed on the mastoid process behind the ear. The tuning fork is not placed near the external meatus of each ear or under the bridge of the nose.

During the history a client reports a blockage in the upper portion of the nasal passage. Which of the following would the nurse expect as a prominent symptom?

Inability to smell Explanation: Receptors for cranial nerve I (olfactory) are located in the upper part of the nasal cavity and septum. Blockage would decrease the ability to smell. A decreased ability to taste would be associated with an upper respiratory infection or lesion of the facial nerve. Difficulty hearing or occasional dizziness are associated with ear problems.

The RN should intervene and further educate the nursing assistant when observing which action?

Independently pulling an immobile client up in bed Explanation: Friction/shear forces are risks to breaks in skin integrity that can occur when pulling a client up in bed alone. The nursing assistant needs to ask for assistance when repositioning an immobile client. Assisting with feeding or ambulating, and using pillows under bony prominences to prevent pressure ulcers are all appropriate nursing assistant tasks.

The client states her husband died a few months ago and she has not been the same since. Which nursing diagnosis is most appropriate?

Ineffective coping Explanation: Ineffective coping would be most appropriate. Anticipatory grieving occurs prior to change. There is no evidence of fear or mental status change.

The nurse has completed a focused ear and hearing assessment and gathered the following data: the client speaks very softly, denies hearing loss, and has never had and cannot afford additional hearing tests; the client fails the whisper test. Which nursing diagnosis would be most appropriate?

Ineffective health maintenance related to denial of hearing problem and inadequate resources for additional testing Explanation: A nursing diagnosis of ineffective health maintenance would be most appropriate based on the data. There is nothing to suggest that the client is having difficulty with social interaction. A soft speaking voice does not indicate a problem with impaired verbal communication. The client has a problem, so a health promotion diagnosis of readiness for enhanced communication would be inappropriate.

Which of the following would be most important for the nurse to do when assessing a client's blood pressure?

Inflate the cuff 30 mmHg above where the radial pulse disappears. Explanation: The nurse should inflate the cuff by pumping the bulb to about 30 mmHg above the point at which the radial pulse disappears. Doing so will help to avoid missing an auscultatory gap. The nurse would palpate the pulsations of the brachial artery, hold the client's arm slightly flexed with the palm up, and deflect the cuff about 2 mmHg per second.

The nurse is performing an ear assessment of an adult client. Which action constitutes the correct procedure for using an otoscope when examining the client's ears?

Inserting the speculum down and forward into the ear canal Explanation: The nurse should insert the speculum gently down and forward into the canal. Using the dominant hand, the nurse should position the hand holding the otoscope against the client's head or face. The largest speculum that fits comfortably into the client's ear canal is used.

Which of the following denotes the correct procedure for using an otoscope when examining the ears of a 32-year-old client?

Inserting the speculum down and forward into the ear canal Explanation: The nurse should insert the speculum gently down and forward into the canal. Using the dominant hand, the nurse should position the hand holding the otoscope against the client's head or face. The largest speculum that fits comfortably into the client's ear canal is used.

A child presents to the health care facility with new onset of a foul smelling, purulent drainage from the right nare. The mother states no other signs of an upper respiratory tract infection are present. What is an appropriate action by the nurse?

Inspect the nostrils with an otoscope Explanation: Because the drainage is unilateral, the most likely cause is a foreign body obstruction. He nurse should inspect the nostrils for patency and the presence of a foreign body. It is not a normal finding in children to have unilateral foul smelling drainage from the nose. This child will not need an antibiotic, so the nurse does not need to assess for allergies to medication. Blowing the nose may or may not dislodge the object and may cause further trauma to the nare.

What assessment technique is performed for every body part and body system?

Inspection Explanation: Inspection is the one technique that is performed for every body part and body system. Other techniques are not used for every body part and system.

What does examination of the skin involve? Select all that apply.

Inspection Palpation Explanation: Examination of the skin involves inspection and palpation. It does not generally involve a nutrition assessment, percussion, or auscultation.`

A nurse is beginning the physical examination of an elderly man with chronic obstructive pulmonary disease. In which order should the nurse implement the four physical assessment techniques with this client?

Inspection, palpation, percussion, auscultation Explanation: Four basic techniques must be mastered before you can perform a thorough and complete assessment of the client. These techniques are inspection, palpation, percussion, and auscultation. Inspection precedes palpation, percussion, and auscultation because the latter techniques can potentially alter the appearance of what is being inspected.

In which order should a nurse implement the four physical assessment techniques when initiating a health assessment?

Inspection, palpation, percussion, auscultation Explanation: Inspection is the first physical assessment technique that a nurse should implement. This prevents altering the appearance of structures that may distract the nurse from completing a focused observation.

A nurse is assessing a client who is exhibiting decorticate posturing. Which of the following would the nurse observe?

Internally rotated lower extremities Explanation: Decorticate posturing is manifested by the hands drawn up to the chest and internally rotated lower extremities. Decerebrate posturing is manifested by adducted and extended upper extremities, pronated forearms, and flexed hands along the side of the body.

A nurse is assessing the pain of a client who has had major surgery. The client also has been experiencing depression. Which of the following principles should guide the nurse's assessment of a client's pain?

It is likely that the client's pain rating will be influences by his emotional state. Explanation: Although pain perception is poorly understood, studies have shown that the emotional status of a client, such as depression or anxiety, directly affects the level of pain perceived and thus reported by clients. The nurse needs to be conscious of and able to react to the possibility that a client with depression is experiencing more pain, or less pain, than he or she is reporting.

Mrs. Hill is a 28-year-old woman of African ancestry with a history of systemic lupus erythematosus (SLE). She has noticed a raised dark red rash on her legs. When the nurse presses on the rash, it doesn't blanch. What would the nurse tell the client regarding her rash?

It is likely to be related to her lupus. Explanation: A "palpable purpura" is usually associated with a vasculitis. This is an inflammatory condition of the blood vessels often associated with systemic rheumatic disease. It can cut off circulation to any portion of the body and mimic many other diseases. While allergic and chemical exposures may be a possible cause of the rash, this client's SLE should make the nurse consider vasculitis.

A nurse asks a client the following question: "What do you do if you have pain?" The nurse is assessing which of the following aspects of cognitive function?

Judgment Explanation: Asking a client about what he or she does or would do if he or she has pain evaluates a client's judgment. Asking about the client's name, time, and place evaluates his or her orientation. Asking a client to compare and contrast things evaluates abstract reasoning. Asking the client about recent and past events evaluates memory.

A nurse asks a client the following question: "What do you do if you have pain?" The nurse is assessing which of the following?

Judgment Explanation: Asking a client about what he or she does or would do if he or she has pain evaluates a client's judgment. Asking about the client's name, time, and place evaluates his or her orientation. Asking a client to compare and contrast things evaluates abstract reasoning. Asking the client about recent and past events evaluates memory.

A nurse must examine the rectum of a woman who has complained of bleeding from the anus and pain on defecating. Which of the following positions would be most appropriate for the client?

Knee-chest Explanation: The knee-chest position is useful for examining the rectum. In this position, the client kneels on the examination table with the weight of the body supported by the chest and knees. In the prone position, the client lies down on the abdomen with the head to the side. The prone position is used primarily to assess the hip joint. In the supine position, the client lies down with the legs together on the examination table. This position allows the abdominal muscles to relax and provides easy access to peripheral pulse sites. Areas assessed with the client in this position may include the head, neck, chest, breasts, axillae, abdomen, heart, lungs, and all extremities. In the dorsal recumbent position, the client lies down on the examination table or bed with the knees bent, the legs separated, and the feet flat on the table or bed. Areas that may be assessed with the client in this position include the head, neck, chest, axillae, lungs, heart, extremities, breasts, and peripheral pulses.

A client frequently experiences dry, irritated eyes. These findings are consistent with a problem in what part of the eye?

Lacrimal apparatus Explanation: The lacrimal apparatus (which consists of the lacrimal gland, punctum, lacrimal sac, and nasolacrimal duct) protects and lubricates the cornea and conjunctiva by producing and draining tears.

A nurse is teaching a group of 5th grade children about characteristics of the skin. Which of the following should she mention? Select all that apply.

Largest organ of the body Protects against damage to the body from sunlight Helps make vitamin D in the body Aids in maintaining body temperature Explanation: The skin is the largest organ of the body. The skin is a physical barrier that protects the underlying tissues and organs from microorganisms, physical trauma, ultraviolet radiation, and dehydration. It plays a vital role in temperature maintenance, fluid and electrolyte balance, absorption, excretion, sensation, immunity, and vitamin D synthesis. The heart, not the skin, circulates blood throughout the body. The digestive system, not the skin, is involved in digestion of food.

A client opens the eyes and answers questions however falls back asleep within seconds. How should the nurse document this assessment finding?

Lethargy Explanation: Opening the eyes, answering questions, and falling back asleep describes lethargy. Being completely unresponsive to all stimuli with the eyes closed describes a coma. Being awakened with vigorous or painful stimuli describes stupor. Opening the eyes to loud voices, responding slowly with confusion, and being unaware of the environment describes obtunded.

Which finding, if noted when inspecting a client's mouth, would require immediate follow-up?

Leukoplakia Explanation: Leukoplakia is a precancerous lesion that requires immediate follow-up. Although thrush, which indicates a candidal infection; Koplik spots, which are an early sign of measles; and canker sores, which are associated with adrenocortical insufficiency, are abnormal findings, the evidence of leukoplakia is serious and needs immediate evaluation.

What physical assessment technique should a nurse use to obtain a pulse on a client?

Light palpation Explanation: The nurse should use the light palpation technique to check the pulse of the client. Moderate and bimanual palpation is used to note the size, consistency, and mobility of the structures that are palpated. Deep palpation enables the nurse to feel very deep organs or structures that are covered by thick muscles.

A nurse needs to obtain a pulse on a client. Which physical assessment technique should the nurse use?

Light palpation Explanation: The nurse should use the light palpation technique to check the pulse of the client. Moderate and bimanual palpations are used to note the size, consistency, and mobility of the structures that are palpated. Deep palpation enables the nurse to feel very deep organs or structures that are covered by thick muscles.

During the physical examination of your client you auscultate the sound of the client's breathing. What area of the client are you assessing?

Lungs Explanation: To assess the client's breathing sounds, the nurse auscultates the lungs using the stethoscope.

A 32-year-old white woman comes to the clinic complaining of overwhelming sadness. She says for the past 2 months she has had crying episodes, difficulty sleeping, and problems with overeating. She says she used to go out with friends from work but now she just wants to go home and be alone. She also thinks that her work productivity has been dropping because she just is too tired to care or concentrate. She denies any feelings of guilt or suicidal ideation. She states that she has never felt this way in the past. She denies any recent illness or injuries. Past medical history consists of an appendectomy when she was a teenager; otherwise, she has been healthy. She is single and works as a clerk in a medical office. She denies tobacco, alcohol, or illegal drug use. Her mother has high blood pressure, and her father has a history of mental illness. Examination reveals a woman who appears her stated age and seems sad. Her facial expression does not change during conversation, and she makes little eye contact. She speaks so softly that the nurse cannot always understand her. Her thought processes and content seem unremarkable. What type of mood disorder is most consistent with these findings?

Major depressive episode Explanation: Major depression occurs in a person with a previously normal state of mood. Symptoms often consist of a combination of sadness, decreased interest, sleeping problems (insomnia or hypersomnia), eating problems (decreased or increased appetite), feelings of guilt, decreased energy, decreased concentration, psychomotor changes (retardation or agitation), and preoccupation with thoughts of death or suicide. There must be five symptoms for a diagnosis of major depression. This client has all five: (1) sadness, (2) trouble sleeping, (3) overeating, (4) fatigue, (5) difficulty with concentration, and (6) no interest in doing things.

A 23-year-old ticket agent is brought in by her husband because he is concerned about her recent behavior. He states that for the last 2 weeks she has been completely out of control. She hasn't showered in days, stays awake most of the night cleaning their apartment, and has run up more than $5,000 on their credit cards. While he is talking the client interrupts him frequently, declares this is all untrue, and says she has never been so happy and fulfilled in her whole life. She speaks very quickly, changing the subject often. After a longer than normal interview, the nurse learns that the client has had no recent illnesses or injuries. Her past medical history is unremarkable. Both her parents are healthy, but the husband has heard rumors about an aunt with similar symptoms. The client and her husband have no children. She smokes one pack of cigarettes a day (although she has been chain smoking in the last 2 weeks), drinks four to six times a week, and smokes marijuana occasionally. She is very loud and outspoken. Physical examination findings are unremarkable. Which mood disorder does she most likely have?

Manic episode Explanation: Mania consists of a persistently elevated mood for at least 1 week with symptoms such as inflated self-esteem, decreased need for sleep, pressured speech, racing thoughts, and involvement in high-risk activities (e.g., drug use, spending sprees, indiscriminate sexual activity). In this case, the client has racing thoughts and pressured speech, a decreased need for sleep, and engagement in high-risk activities (spending sprees).

The nurse is assessing the face of a client with a diagnosis of Parkinson's disease. What would the nurse most likely assess?

Mask-like expression Explanation: A client with Parkinson's disease often exhibits a masklike face. A sunken face with depressed eyes and hollow cheeks is typical of cachexia. Drooping of one side may suggest a stroke or Bell's palsy. Asymmetry of the earlobes occurs with parotid gland enlargement from an abscess or tumor.

A nurse recognizes that it is best to begin the objective data collection with which procedure?

Measure the client's vital signs, height, and weight Explanation: It is important to begin the assessment with less intrusive procedures such as vital signs and height & weight. These nonthreatening/nonintrusive procedures allow the client to feel more comfortable with the nurse and ease anxiety. Once a trusting relationship is established, the nurse can proceed in a systematic approach to ensure that all body systems are fully examined. Auscultation of all body systems is not an acceptable approach to a comprehensive assessment. The initial assessment data can be collected while the client is still dressed.

When examining the eye with an ophthalmoscope, where would the nurse look to visualize the optic disc?

Medially toward the nose Explanation: Follow the blood vessels as they get wider. Follow the vessels medially toward the nose and look for the round yellowish orange structure which is the optic disc.

A client presents to the emergency department with reports of neck pain and a sudden onset of a headache. Upon examination, the nurse finds that the client has an increased temperature and nuchal rigidity. The nurse recognizes these findings as most likely to be caused by what condition?

Meningeal inflammation Explanation: Meningeal inflammation is a likely cause of this condition which manifests as sudden headache, neck pain with stiffness, and fever. Migraine headaches are accompanied by nausea, vomiting, and sensitivity to noise or light and not by fever and neck stiffness. Trigeminal neuralgia is manifested by sharp, shooting, piercing facial pains that last from seconds to minutes. Parkinson's disease is not manifested by headache and neck pain.

A client presents to the emergency department with reports of neck pain and a sudden onset of a headache. Upon examination, the nurse finds that the client has an increased temperature and neck stiffness. The nurse recognizes these findings as most likely to be caused by what condition?

Meningeal inflammation Explanation: Meningeal inflammation is a likely cause of this condition, which manifests as sudden headache, neck pain with stiffness, and fever. Migraine headaches are accompanied by nausea, vomiting, and sensitivity to noise or light, not by fever and neck stiffness. Trigeminal neuralgia is manifested by sharp, shooting, piercing facial pains that last from seconds to minutes. Parkinson's disease is not manifested by headache and neck pain.

A nurse is caring for a client admitted with neck pain. The client is febrile. What is the most likely medical diagnosis for this client?

Meningitis Explanation: Neck pain associated with fever and headache may signify serious illness such as meningitis and should be carefully evaluated.

A client is being assessed for a headache. Symptoms include throbbing and severe pain lasting for the last 8 hours. The client also has a history of vomiting with the headache. What type of headache could these findings indicate?

Migraine Explanation: A throbbing, severe, unilateral headache that lasts 6 to 24 hours and is associated with photophobia, nausea, and vomiting suggests migraine. The scenario does not indicate tension, cluster, or benign headaches.

A client reports right-sided temporal headache accompanied by nausea and vomiting. A nurse recognizes that which condition is likely to produce these symptoms?

Migraine headache Explanation: Migraine headaches are usually located around the eyes, temples, cheeks, and forehead. They are often accompanied by nausea and vomiting. Bell's palsy is a one sided facial paralysis caused by inflammation of the facial nerve. A tension headache usually presents with stress, anxiety, or tension and is located in the frontal, temporal, or occipital region. Temporal arteritis produces pain around the temple but no nausea or vomiting.

Which risk factor for traumatic brain injury should a nurse include in a discussion about prevention for a group of adolescents?

Modes of transportation are the leading cause Explanation: All modes of transportation, such as motor vehicle & bicycles, are the leading cause of traumatic brain injuries for people age 5 to 64 years. Males have twice the risk of females. Firearm injuries are high in the violence category and two thirds are suicidal in intent. Fall occur most frequently in the over 65 years of age population.

A nurse in the emergency department assesses a client's pupillary reaction and observes pinpoint pupils. The nurse interprets this finding as suggesting which of the following?

Narcotic use Explanation: Pinpoint pupils suggest narcotic use or brain damage. Hyphema would suggest recent eye trauma. Dilated and fixed pupils typically result from central nervous system injury, circulatory collapse, or deep anesthesia.

The nurse inspects a client's mouth and notes the presence of a bifid uvula. The nurse understands that this finding is most common in which ethnic group?

Native Americans Explanation: A bifid uvula is a common assessment finding in Native Americans.

The nurse asks the client to perform the action pictured. What is the nurse assessing?

Near vision Explanation: The client is using the Jaeger chart which is used to assess near vision. The Snellen chart is used to assess distant vision. The nurse would not assess intraocular pressure. Ishihara cards are used to assess color discrimination.

During the review of systems, a client reports dizziness, tingling, and mood changes. In which area should the nurse document this information?

Neurologic Explanation: Dizziness, tingling, and mood changes would be documented under neurologic. Nervousness, tension, depression, memory change, and suicide attempts should be documented under psychiatric. This information is not appropriate to document under cardiovascular or fluid and electrolytes.

When teaching a class of school-age children about hygiene, the nurse should include which information about the ears?

Never put anything smaller than your elbow in your ears. Explanation: The nurse should reinforce proper cleaning techniques such as cleaning the bowl of the helix and never introducing anything into the external auditory canal. An elbow will not fit into an ear canal; therefore stating not to put anything smaller than an elbow into the ears, eliminates putting anything into the external ear canal. It's also a fun way to educate school age children. Cotton tipped applicators can cause complications and should not be used to clean the ears. An increased amount of earwax, not decreased, can lead to conductive hearing loss.

While inspecting the tympanic membrane, the nurse notes a pearly gray and shiny appearance. The nurse would interpret this finding as which of the following?

Normal tympanic membrane Explanation: The tympanic membrane is normally a pearly gray color with a shiny appearance. White spots would indicate scarring. <wbr />A yellowish bulging membrane would suggest serous otitis media; a red bulging membrane would suggest acute otitis media.

A client complains of a unilateral headache near the scalp line and double vision. The nurse palpates the space above the cheekbone near the scalp line on the affected side, and the client complains of tenderness on palpation. What is the nurse's next action?

Notify the healthcare provider immediately. Explanation: Temporal arteritis is a painful inflammation of the temporal artery. Clients report severe unilateral headache sometimes accompanied by visual disturbances. This condition needs immediate care. A biopsy may be necessary for diagnosis; however the healthcare provider immediately. The temporal artery pulse can be palpated; but the carotid artery pulses should never be palpated simultaneously so that the client does not pass out from lack of blood flow to the brain.

When performing an ophthalmoscopic exam, a nurse observes a round shape with distinct margins. The nurse would document this as which of the following?

Optic disc Explanation: The optic disc is round to oval with sharp, well-defined borders. The physiologic cup appears on the optic disc as slightly depressed and a lighter color than the disc. Arteriole retinal vessels appear bright red, and venules appear darker red and larger, with both progressively narrowing as they move away from the optic disc. The fovea is a small area of the retina that provides acute vision.

What steps are involved in the client-to-client transmission of pathogens? (Select all that apply.)

Organisms are transferred from the client to the nurse's hands The nurse's contaminated hands come into direct contact with another client Organisms are present in the client's immediate environment Explanation: Patient-to-client transmission of pathogens requires five sequential steps: (1) Organisms are present on a client's skin or immediate environment; (2) Organisms are transferred from the client to the nurse's hands; (3) Organisms survive on the nurse's hands for at least several minutes; (4) The nurse omits or performs inadequate or inappropriate hand hygiene; (5) The nurse's contaminated hands come into direct contact with another client or environment in direct contact with the client. The use of alcohol-based hand sanitizer breaks the five-step process. Organisms can live on the nurse's hands for more than 1 minute if not cleansed appropriately.

When the mental health nurse ask the client "Do you recall what month and year this is?" The nurse is assessing which part of the mental status examination?

Orientation Explanation: One of the most basic assessments of cognitive function is the client's orientation to person, place, and time. Judgment may be viewed as the action-oriented counterpart to insight. To assess abstract reasoning the nurse may ask the client to describe the meaning of well-known proverbs. Insight is the cognitive process of understanding.

A group of students is reviewing material about assessing mental status. The students demonstrated understanding of the material when they identify which of the following as a cognitive ability to be assessed?

Orientation Explanation: Cognitive abilities include orientation, concentration, recent and remote memory, abstract reasoning, judgment, visual perception, and constructional ability. Posture, speech, and thought processes are components of a comprehensive mental status examination.

As part of assessing the client's level of consciousness, the nurse asks questions related to person, place, and time. Which of these statements is true?

Orientation to time is usually lost first and orientation to person is usually lost last. Explanation: When assessing orientation to time, place, and person remember that orientation to time is usually lost first and orientation to person is usually lost last.

The nurse documents findings from the client's Mini-Mental State Examination. The following information will be documented as a result of this test.

Orientation, memory, and cognitive function. Explanation: Cognitive abilities include orientation, concentration, recent and remote memory, abstract reasoning, judgment, visual perception, and constructional ability.

What is the common channel for the respiratory and digestive systems?

Oropharynx Explanation: The oropharynx is the common channel for the respiratory and digestive systems. The frenulum is part of the tongue. The nares are part of the nose.

You are a pediatric nurse caring for a child who has been brought to the clinic with otitis externa. What assessment finding is characteristic of otitis externa?

Pain on manipulation of the auricle Explanation: Tophi are deposits of uric acid crystals and are generally painless; they are a common physical assessment finding in clients diagnosed with gout. Cerumen is a normal finding during assessment of the ear canal. Its presence does not necessarily indicate that inflammation is present. Pain when the nurse pulls gently on the auricle in preparation for an otoscopic examination of the ear canal is a characteristic finding in clients with otitis externa. Air bubbles in the middle ear may be visualized with the otoscope; however, these do not indicate a problem involving the ear canal. Aural tenderness or pain is not usually associated with middle ear disorders.

During a health history, a client reports complaints of headaches. What would lead the nurse to suspect that the client is experiencing cluster headaches?

Pain radiating from eye to temporal region Explanation: Cluster headaches are typically localized in the eye and orbit and radiate to the facial and temporal regions. Throbbing severe pain, reports of ringing in the ears prior to the headache, and sensitivity to light suggest migraine headache.

In interviewing a client about his heart rate, the nurse asks whether he has noticed any alteration to his heartbeat. The client responds that he sometimes feels his heart race even when he has not been exerting himself physically. This alteration is known as which of the following?

Palpitation Explanation: An alteration in heartbeat felt by a client is called a palpitation and can be caused by various circumstances including thyroid dysfunction, medication reaction, or alteration in fluid volume. Dyspnea is difficulty breathing. Pulse pressure is the difference between systolic and diastolic blood pressures. Apical beats are simply the beats of the heart palpated directly over the apex of the heart, on the chest.

While assessing an adult client, the nurse observes an elevated, palpable, solid mass with a circumscribed border that measures 0.75 cm. The nurse documents this as a

Papules are elevated, palpable, solid masses smaller than 1 cm. Plaques are greater than 1 cm and may be coalesced papules with a flat top.

A nurse observes the gait of an elderly client admitted for surgery. The client's gait is stiff with rigid movements. The nurse should ask this client questions about which disease?

Parkinson's disease Explanation: A stiff, shuffling, rigid gait is seen in persons with Parkinson's disease due to the destruction of dopamine receptors in the brain that maintain balance between contraction and relaxation of the muscles. COPD clients have no problems with gait except that activity makes them short of breath. Lordosis of the spine is seen in pregnant women and occurs in the lumbar area of the spine due to the weight of the developing fetus. Multiple sclerosis causes muscle weakness, not rigidity.

Upon inspection of a client with reports of a fever, the nurse notices that the client's earlobes are asymmetrical in appearance. The nurse recognizes that the most common cause for the asymmetry of the earlobes is what condition?

Parotid enlargement Explanation: Earlobe asymmetry can be due to parotid gland enlargement caused by an abscess or tumor. Bell's palsy is a neurologic condition that may cause drooping of one side of the face. Acute pharyngitis causes swelling in the throat, which is not usually visible on the outside of the face. Thyroid enlargement affects the neck and has no effect on the symmetry of the earlobes.

The client is brought to the clinic by his son, who states, "My father just doesn't seem to be able to function as well as he used to." When assessing this client the nurse is aware that she will be a what?

Patient advocate Explanation: The nurse may assess the change in the client and will be the advocate and detective, determining when the change occurred and what was new in the treatment.

The nursing student demonstrates a need for further teaching when she states which of the following?

Patients do not need to understand their problems. Explanation: It is essential for the client to understand the problem so that treatment can be properly implemented. If the client is not coherent, it is proper to consult with the family or significant other or even other health care workers. Validation is also important with the client who has a collaborative problem or who requires a referral.

The nurse would document driving with car seatbelt fastened, bicycling with properly-fitted helmet, and installing a smoke detector in a vacation home in the client's health history under which of the following?

Personal and social history Explanation: Health-maintenance practices such as immunizations, screening tests, lifestyle issues, and home safety are components of the client's personal and social history.

While inspecting the skin of an older adult client, the nurse notes multiple small, flat, reddish-purple macules. The nurse should recognize the presence of which of the following?

Petechiae Explanation: Petechiae are small, round, red or purple macules that are secondary to blood extravasation. Purpura refers to hemorrhagic disease that produces ecchymoses and petechiae. Ecchymosis refers to round or irregular macular lesions that are larger than petechiae and are variable in color. A cherry angioma is a papular round red or purple lesion.

The nurse has completed a focused assessment of a client's mouth, nose, and throat. Which finding would the nurse interpret as being normal?

Pinkish, spongy soft palate Explanation: The soft palate is expected to be pinkish, soft, spongy, and movable. A negative red glow on transillumination of the sinuses indicates that a sinus is filled with pus or fluid. Nasal mucosa that is pale pink and swollen suggests allergies. Tonsils greater than 1+ are considered abnormal.

A nurse has taught a group of older adults about the high incidence and prevalence of macular degeneration. What health promotion and prevention activity should the nurse encourage these clients to perform?

Post an Amsler grid in their home and perform the test on a regular basis Explanation: The Amsler test is used to identify the early symptoms of macular degeneration. This does not involve the use of a Snellen chart. Sodium limitation is beneficial to overall health but does not specially address the risk factors for macular degeneration. Rinsing the eyes is not necessary or relevant.

The nurse observes a middle-aged colleague fully extending her arm to read the label on a vial of medication. Which of the following age-related changes is the nurse likely to have observed?

Presbyopia Explanation: Presbyopia denotes an age-related deficit in close vision. It is less likely that cataracts, macular degeneration, or loss of convergence underlie the colleague's visual changes.

The nurse would pursue additional assessment and evaluation of an older adult client with diabetes upon assessing which of the following?

Pressure ulcer Explanation: An older adult client most likely would have thin, fragile skin, which can result in easy breakdown and slower wound healing. Evidence of a pressure ulcer would require additional assessment. A cherry angioma usually is not clinically significant. A cutaneous horn or seborrheic keratosis is considered a common skin variation.

The nurse wears gloves for which of the following purposes? Select all that apply.

Prevent transmission of flora from client to client. Limit exposure to body fluids and secretions Explanation: The nurse wears gloves to prevent transmission of flora from client to client, prevent exposure to body fluids and secretions, decrease the risk of the nurse acquiring infection from the client, and reduce contamination of the hands of the nurse.

During the interview process, the nurse obtains what type of data from the client?

Primary Explanation: Nurses collect primary data from clients themselves. Secondary data come from family and medical records. Objective data are data observed. Oral data is a form of data obtained through conversation.

The client is being interviewed upon arrival in the Emergency Department. When collecting subjective data from the client, the nurse is obtaining what other type of data from the client?

Primary Explanation: Subjective data given by the client are considered primary data. Charts and family are sources of secondary data, while objective data are based upon tests, vital signs, and examinations. At present, no data are called tertiary.

As a nursing student you learn that mastering all the components of the comprehensive history provides what?

Proficiency Explanation: Mastery of all the components of the comprehensive history provides proficiency and the ability to select the elements most pertinent to the client encounter.

A nurse is assessing the eyes of a 3-year-old child. Which finding would the nurse document as normal?

Pseudostrabismus Explanation: In young children, pseudostrabismus would be considered a normal finding due to the epicanthal folds. Tropia, nystagmus, and exotropia would be abnormal findings.

A young man comes to the clinic with an extremely pruritic rash over his knees and elbows, which has come and gone for several years. It seems to be worse in the winter and improves with some sun exposure. Examination reveals scabbing and crusting with some silvery scales. The nurse also notices small "pits" in the nails. What would account for these findings?

Psoriasis Explanation: This is a classic presentation of plaque psoriasis. Eczema is usually over the flexor surfaces and does not scale, whereas psoriasis affects the extensor surfaces. Pityriasis usually is limited to the trunk and proximal extremities. Tinea has a much finer scale associated with it, almost like powder, and is found in dark and most areas.

What action should the nurse implement using an otoscope when assessing the ear of an adult client?

Pull the auricle out, up, and back Explanation: The nurse should pull the auricle out, up, and back to straighten the external auditory canal. This is because the external auditory canal is S-shaped in the adult. The outer part of the canal curves up and back, and the inner part of the canal curves down and forward. The nurse should choose the largest speculum that fits comfortably into the client's ear. The nurse should hold the speculum in the dominant hand and insert the speculum gently down and forward.

The nurse assesses the amplitude of the client's radial pulse and finds it to be weak and diminished. Which of the following scores should the nurse record?

Pulse amplitude of 0 means that it is absent, 1+ that it is weak and diminished (easy to obliterate), 2+ that it is normal (can be obliterated with moderate pressure), and 3+ that it is bounding (unable to obliterate or requires firm pressure).

When testing the near reaction, an expected finding includes which of the following?

Pupillary constriction on near gaze; dilation on distant gaze Explanation: During accommodation, pupils constrict with near gaze and dilate with far gaze.

When assessing a client's pulse, the nurse should be alert to which of the following characteristics?

Rate, rhythm, amplitude and contour, and elasticity. Explanation: Several characteristics should be assessed when measuring the radial pulse: rate, rhythm, amplitude and contour, and elasticity.

A nurse is assessing the blood pressure on an obese woman. What error might occur if the cuff used is too narrow?

Reading is erroneously high. Explanation: The bladder of the cuff should enclose at least two-thirds of the adult limb. If the cuff is too narrow, the reading could be erroneously high because the pressure is not being transmitted evenly to the artery.

A comprehensive health history includes which components? Select all that apply.

Reason for seeking care History of present illness Past health history Explanation: Usually the nurse collects demographical data first and then elicits from clients a complete description of their reason for seeking care, because that information usually is most important. The nurse collects information about the present illness by beginning with open-ended questions and having clients explain symptoms. A complete description of the present illness is essential to an accurate diagnosis.

A nurse is assessing the mouth of an older client. Which of the following findings is common among older adults?

Receding and ischemic gums Explanation: The gums recede, become ischemic, and undergo fibrotic changes as a person ages. A bifid uvula is a common finding in Native Americans, not among older adults. Brown spots on the chewing surface of teeth is an indication of tooth decay and is not associated with aging per se, nor are enlarged palatine tonsils, which are an indicator of tonsillitis.

The nurse notes otitis media with effusion in the left ear of a 3-year-old child. Which assessment data is consistent with otitis media with effusion?

Redness and bulging of the eardrum Explanation: Redness and bulging of the eardrum are characteristic of otitis media with effusion. Clear or bloody discharge occurs with rupture of the tympanic membrane. Dense white patches on the tympanic membrane are noted with scarring of the tympanic membrane.

After conducting a health history, the nurse decides to perform the assessment shown. What finding did the nurse use to make this clinical determination?

Reduced hearing in one ear Explanation: The Weber test is used to determine unilateral hearing loss. With conductive hearing loss, the client reports hearing the tuning fork sound better in the poor ear. With sensorineural hearing loss, the client hears the sound better in the good ear. The Weber test is not used to assess a sore throat. There are many reasons for a rigid tympanic membrane. A Weber test is used to test for hearing only. This test will not help diagnose the reason for edematous neck lymph nodes.

A nurse is providing care at an inner-city shelter, and a man who frequents the shelter presents with a significant frontal growth that is located midline at the base of his neck. The nurse should recognize the need for what referral?

Referral for further assessment of thyroid function Explanation: A goiter (an enlarged thyroid gland) may appear as a large swelling at the base of the neck. This growth is not suggestive of impaired cranial nerve or lymphatic function, and it does not normally impair swallowing ability.

What is the nurse assessing when asking the client, "What things seem to make it better?"

Relieving/Exacerbating factors Explanation: Assess for relieving factors by asking, "What seems to make it better?" or "What have you tried to make it go away?"

The ICU nurse is caring for a trauma victim whose status is critical. On assessment, the nurse notes uremic frost along the client's hairline. What would this indicate to the nurse?

Renal failure Explanation: Uremic frost is a sign of marked renal failure. This appearance results from precipitation of renal urea and nitrogen waste products through sweat onto the skin. Uremic frost is not related to cardiovascular failure, hepatic failure, or respiratory failure.

A client has come to the physician's office several times in the last month with a black eye, bruises, and lacerations on the lower extremities. The client always explains having fallen and tripped. The nurse suspects abuse. The next step should be to:

Report the findings to a supervisor. Explanation: When abuse is suspected, nurses are obligated to report their concerns to a supervisor and obtain assistance from social services for further assessment. It is not in the nurse's scope of practice to call social services directly, and the nurse should never call the police independently. Also confronting the client will only isolate her and make her more uncomfortable.

During a physical examination the nurse performs the action shown. What assessment is the nurse performing?

Rinne test Explanation: The first part of the Rinne test is to place the handle of a vibrating tuning fork on the mastoid process. This is assessing hearing through bone conduction. The Weber test is conducted by placing the handle of a vibrating tuning fork on the top of the head. It is used to differentiate the cause of unilateral hearing loss. Auditory acuity is determined through an audiogram in a soundproof room. Vestibular function or equilibrium is assessed through the Romberg test.

A 19-year-old college student, Todd, comes to the clinic with his mother, who is concerned that there is something seriously wrong with him. She states that for the past 6 months, her son's behavior has become peculiar, and that he has flunked out of college. Todd denies any recent illness or injuries. His past medical history is remarkable only for a broken foot. His parents are healthy. He has a paternal uncle who had similar symptoms in college. The client admits to smoking cigarettes and drinking alcohol. He also admits to marijuana use but not in the last week. He denies use of any other substances and feelings of depression or anxiety. The nurse does a complete physical examination, which is essentially normal. When the nurse questions the client about how he is feeling, he says that he is worried that his software for creating a better browser has been stolen. He says that he has seen a black van in his neighborhood at night, and he is sure that it is full of computer programmers stealing his work through special gamma waves. The nurse asks why Todd believes they are trying to steal his programs. He replies that the programmers have been telepathing their intents directly into his head. He says he hears these conversations at night, so he knows this is happening. What psychotic disorder is most consistent with Todd's history and physical examination findings?

Schizophrenia Explanation: Onset of schizophrenia generally happens in the late teens to early 20s. It often is seen in other family members. Symptoms must be present for at least 6 months and must have at least two features of (1) delusions (thieves are stealing his programs), (2) hallucinations (technicians sending telepathic signals), (3) disorganized speech, (4) disorganized behavior, and (5) negative symptoms such as a flat affect. The catalysts of delirium and substance ingestion that denote a psychotic disorder due to medical illness and substance-induced psychotic disorder are absent. Generalized anxiety is not present.

Which of the following is a symptom of the eye?

Scotomas Explanation: Scotomas are specks in the vision or areas where the client cannot see; therefore, this is a common and concerning symptom of the eye. Tinnitus is a ringing in the ears, dysphagia is difficulty swallowing, and rhinorrhea is a "runny nose."

Susanne is a 27-year-old woman who has had headaches, muscle aches, and fatigue for the last 2 months. The nurse has completed a thorough history, examination, and laboratory workups, the results of which are normal. What would the next action be?

Screening for depression Explanation: Although the nurse may consider referrals to help with diagnosis and treatment of this client, screening is a time-efficient way to recognize depression. This will allow her to be treated more expediently. The nurse may tell the client that no answer is clear yet, but also that he or she will not stop investigating until the client has gotten the help she needs. Research has shown that health care providers routinely fail to screen for depression.

The nurse is admitting a new client to the unit. While reviewing old records of the client, the nurse knows that the data being gathered are what kind of data?

Secondary Explanation: Charts and family members are considered secondary data sources. The client is the source of primary data. Subjective data are data provided to the nurse by the client; objective data are data that the nurse observes.

The nurse is assessing an older adult client a hospice unit. The client cannot speak or communicate, but the client's daughter is there and answers all the questions as best as she can. What type of data source is the daughter?

Secondary Explanation: Charts and family members are considered secondary data sources. Primary data would be directly from the client. Subjective data are based on the signs and symptoms that the client reports; they may not be perceived by observers.

Jason, a 41-year-old electrician, presents to the clinic for evaluation of shortness of breath, which occurs with exertion and improves with rest. The shortness of breath has been occurring for several months. Initially, it happened only a few times a day with strenuous exertion; however, it has started to occur with minimal exertion and is happening more than 12 times a day. The shortness of breath lasts for fewer than 5 minutes at a time. The client has no cough, chest pressure, chest pain, swelling in his feet, palpitations, orthopnea, or paroxysmal nocturnal dyspnea. Which of the following symptom attributes was not addressed in this description?

Severity Explanation: The interviewer did not record the severity of the symptom, so we have no understanding as to how bad the symptom is for this client. The client could have been asked to rate his pain on a 0 to 10 scale or according to one of the other standardized pain scales available. This allows the comparison of pain intensity before and after an intervention.

During the review of systems, a client reports having difficulty with urination and with establishing an erection. Which additional information should the nurse recognize as the highest priority to assess at this time?

Sexual history Explanation: If the chief complaint involves genitourinary symptoms, include questions about sexual health as part of "expanding and clarifying" the client's story. Lifestyle, medication, and substance use can be contributing factors but gathering a sexual history is the priority action at this time.

A client is assigned a visual acuity of 20/100 in her left eye. Which of the following is true?

She can see at 20 feet what a normal person could see at 100 feet. Explanation: The denominator of an acuity score represents the line on the chart the client can read. In the example above, the client could read the larger letters corresponding with what a normal person could see at 100 feet.

A nurse should assist a client to assume what position to best assess the mouth, nose, and sinuses?

Sitting with the head erect and at the eye level of the nurse Explanation: The nurse should ask the client to assume a sitting position with the head erect and at the eye level of the examiner. Tilting the head backwards and a semi-recumbent position with the chin lifted will make it more difficult to visualize the mouth and nose. The prone position will make transillumination and palpation of the sinuses more difficult for the examiner.

A nurse has gathered the necessary equipment for the physical assessment of an adult client. It would be most appropriate for a nurse to use a centimeter-scale ruler for which measurement?

Skin lesion size Explanation: A centimeter scale rule most likely would be used to measure the size of a skin lesion. A flexible tape measure would be appropriate to measure mid-arm circumference. A vertical scale in inches or meters would be appropriate to measure a client's height. Pupil size is measured in millimeters.

It would be most appropriate for a nurse to use a centimeter-scale ruler for which measurement?

Skin lesion size Explanation: A centimeter scale rule most likely would be used to measure the size of a skin lesion. A flexible tape measure would be appropriate to measure mid-arm circumference. A vertical scale in inches or meters would be appropriate to measure a client's height. Pupil size is measured in millimeters.

A nurse is preparing to perform a physical examination of an obese client who is beginning a diet and exercise program. The physician would like to establish a baseline percent body fat measurement for the client so that the client's progress in reducing body fat can be tracked over time. Which piece of equipment should the nurse anticipate needing for this purpose?

Skinfold calipers Explanation: Skinfold calipers measure skinfold thickness of subcutaneous tissue to aid in establishing a client's percent body fat. A platform scale with height attachment measures height and weight. A metric ruler is used to measure the size of skin lesions. A sphygmomanometer, in conjunction with a stethoscope, is used to measure diastolic and systolic blood pressure.

A client who suffers from arthritis complains of sharp pain in her knees and elbows. The nurse recognizes this is what type of pain?

Somatic Explanation: Pain nociception has various locations. Visceral pain originates from abdominal organs; clients often describe this pain as crampy or gnawing. Somatic pain originates from skin, muscles, bones, and joints; clients usually describe somatic pain as sharp (D'Arcy, 2014). Cutaneous pain derives from the dermis, epidermis, and subcutaneous tissues. It is often burning or sharp, such as with a partial-thickness burn. Referred pain originates from a specific site, but the person experiencing it feels the pain at another site along the innervating spinal nerve.

A nurse measures a client's blood pressure at 174/102 mm Hg. The nurse recognizes this as what classification of blood pressure measurement?

Stage 2 hypertension Explanation: Normal blood pressure: Systolic less than 120 mm Hg and diastolic less than 80 mm Hg. Elevated blood pressure: Systolic between 120 and 129 mm Hg and diastolic less than 80 mm Hg. Stage 1 hypertension: Systolic between 130 and 139 mm Hg or diastolic between 80 and 89 mm Hg. Stage 2 hypertension: Systolic of 140 or greater mm Hg or diastolic of 90 or greater mm Hg.

When asked to assess an area of broken skin on an older adult client in a long-term care facility, the nurse notes a break in the skin erythema and a small amount of serosanguineous drainage over the sacrum. The area appears blister-like. The nurse would interpret this finding as indicating which stage of pressure ulcer?

Stage II Explanation: A stage II ulcer is manifested by a partial-thickness loss of dermis presenting as a shallow open ulcer with a red pink wound bed, without slough; an intact or open/ruptured serum-filled blister; a shiny or dry shallow ulcer without slough or bruising (bruising indicates suspected deep tissue injury). A stage I ulcer is manifested by intact skin with non-blanchable redness of a localized area usually over a bony prominence. The area may be painful, firm, soft, warmer, or cooler as compared to adjacent tissue. A stage III ulcer is manifested by full-thickness tissue loss; possible visible subcutaneous fat with no exposure of bone, tendon, or muscle; possible slough that does not obscure the depth of tissue loss; possible undermining and tunneling. A stage IV ulcer is manifested by full-thickness tissue loss with exposed bone, tendon, or muscle; possible slough or eschar on some parts of the wound bed; often with undermining and tunneling.

A nurse has been asked to assess an older adult resident of a long-term care facility. During assessment of the resident's skin, the nurse notes a break in the skin, erythema, and a small amount of serosanguineous drainage over the resident's sacrum. Inspection reveals that the area appears blister-like. The nurse should interpret this finding as indicating which stage of pressure ulcer?

Stage II Explanation: A stage II ulcer is manifested by a partial-thickness loss of dermis presenting as a shallow open ulcer with a red pink wound bed, without slough; an intact or open/ruptured serum-filled blister; a shiny or dry shallow ulcer without slough or bruising (bruising indicates suspected deep tissue injury). A stage I ulcer is manifested by intact skin with nonblanchable redness of a localized area, usually over a bony prominence. The area may be painful, firm, soft, warmer, or cooler as compared to adjacent tissue. A stage III ulcer is manifested by full-thickness tissue loss; possible visible subcutaneous fat with no exposure of bone, tendon, or muscle; possible slough that does not obscure the depth of tissue loss; possible undermining and tunneling. A stage IV ulcer is manifested by full-thickness tissue loss with exposed bone, tendon, or muscle; possible slough or eschar on some parts of the wound bed; often with undermining and tunneling.

A 15-year-old high school student presents to the emergency department with his mother for evaluation of an area of blood in the left eye. He denies trauma or injury but has been coughing forcefully with a recent cold. He denies visual disturbances, eye pain, or discharge from the eye. On physical examination, the pupils are equal, round, and reactive to light with a visual acuity of 20/20 in each eye and 20/20 bilaterally. There is a homogeneous, sharply demarcated area at the lateral aspect of the base of the left eye. The cornea is clear. Based on this description, what is the most likely diagnosis?

Subconjunctival hemorrhage Explanation: A subconjunctival hemorrhage is a leakage of blood outside of the vessels, which produces a homogenous, sharply demarcated bright red area; it fades over several days, turns yellow, then disappears. There is no associated eye pain, ocular discharge, or changes in visual acuity; the cornea is clear. Many times it is associated with severe cough, choking, or vomiting, which increase venous pressure. It is rare for a serious condition to cause it, so reassurance is usually the only treatment necessary.

A client reports chest pain that occurs with exercise but subsides with rest. The nurse recognizes this as what type of data?

Subjective Explanation: Subjective data includes the following: sensations or symptoms, feelings, perceptions, desires, preferences, beliefs, ideas, values, and personal information collected from the client. This information can be elicited and verified only by the client. Introspection and reflection are not types of data collection but ways in which a nurse can assist a client to work towards changing behaviors. Objective data are obtained by the nurse through observation using the four physical assessment techniques.

The nurse prepares to complete the review of systems with a client. On which area should the nurse focus when completing this part of the assessment?

Symptoms Explanation: Most review of systems questions pertain to symptoms. This section of the assessment does not focus on risk factors, health maintenance, or past medical history.

What finding upon assessment would indicate the client is experiencing shock?

Systolic blood pressure 50 Explanation: A systolic blood pressure of 50 would indicate the client is experiencing shock. All other vital signs, while elevated do not indicate shock

A client has a pulse rate of 28 beats/15 seconds. How should the nurse document this finding?

Tachycardia Explanation: Tachycardia is a heart rate greater than 100 beats/minute. Since the client's heart rate was 28 beats in 15 seconds, the nurse should multiply the rate of 28 x 4 = 112 beats per minute which would be tachycardia. Bradycardia is a heart rate less than 60 beats/minute. There is no evidence to support that the client has skipped beats. A rate of 112 beats/minute is not considered normal.

The nurse is preparing to assess a client's vital signs. Which vital sign should the nurse assess first?

Temperature Explanation: The client's temperature is measured first. Doing so puts the client at ease and causes him or her to remain still for several minutes. This is important because pulse, respiration, and blood pressure are influenced by activity and anxiety.

A nurse is preparing to assess a client's vital signs. In which order should the nurse assess them? (T-P-R-BP)

Temperature, pulse, respirations, and blood pressure Explanation: When assessing the vital signs of the client, the nurse should begin by measuring the clients temperature, to put the client at ease and to quiet the client for better assessment of the remaining vital signs. Pulse, respirations, and blood pressure can be altered by anxiety and activity.

A 29-year-old computer programmer comes to the office for evaluation of a headache. The tightening sensation of moderate intensity is located all over the head. It used to last minutes, but this time it has lasted for 5 days. He denies photophobia and nausea. He spends several hours at a computer monitor/keyboard. He has tried over-the-counter medication; it has dulled the pain, but not taken it away. Based on this description, what is the most likely diagnosis?

Tension

As part of a physical assessment, the nurse performs the confrontation test to assess the client's peripheral vision. Which test result should a nurse recognize as indicating normal peripheral vision for a client using the confrontation test?

The client and the examiner see the examiner's finger at the same time. Explanation: The observation that the client and examiner see the examiner's finger at the same time indicates normal peripheral vision. The client not seeing the examiner's finger or a delay in seeing it indicates reduced peripheral vision. Client's consensual pupils constricting in response to indirect light as well as direct light shown into the client's pupils resulting in constriction are observed when testing the pupils for reaction to light. Eyes converging on an object as it is moved towards the nose is a normal result for accommodation.

An adult client is having his skin assessed. The client tells the nurse he has been a heavy smoker for the last 40 years. The client has clubbing of the fingernails. What does this finding tell the nurse?

The client has chronic hypoxia Explanation: Clubbing of the nails indicates chronic hypoxia. Clubbing is identified when the angle of the nail to the finger is more than 160 degrees. Melanoma does not present with the symptom of clubbing. The scenario described does not give enough information to indicate that the client has COPD or asthma.

The nurse is completing a client's ear assessment. What assessment finding would indicate the need to perform Weber's test?

The client has unilateral hearing loss. Explanation: Unilateral hearing loss is the major indication for Weber's test, which helps distinguish between conductive hearing and sensorineural hearing. Older age, infection, and a history of stroke are not specific indications for this test.

A nurse is assessing an older adult client's risk for pressure ulcers using the Braden Scale for Predicting Pressure Sore Risk. Which aspect of the client's current health status would be reflected in her score on this scale?

The client is consistently incontinent of urine. Explanation: The Braden Scale assesses skin moisture, which is strongly influenced by urinary incontinence. This scale does not specifically address the role of a caregiver, recent surgery, or a vegetarian diet.

The nurse is admitting a 79-year-old man for outpatient surgery. The client has bruises in various stages of healing all over his body. Why is it important for the nurse to promptly document and report these findings?

The client may have been abused. Explanation: Multiple ecchymoses may be from repeated trauma (falls), clotting disorder, or physical abuse.

A nurse is utilizing the Braden Scale for Predicting Pressure Sore Risk during the admission assessment of an older adult client. What assessment parameter will the nurse evaluate when using this scale?

The client's ability to change position Explanation: The Braden Scale appraises the client's level of mobility but does not directly include data related to medications, history of skin disorders, or pigmentation.

A nurse obtains the blood pressure of a client who is uncharacteristically fatigued and who is lying in bed rather than sitting in a chair. The nurse should interpret the client's blood pressure reading in light of what principle?

The client's blood pressure will be slightly lower than standing readings. Explanation: When a client lies down, there is a decrease in peripheral vascular resistance, which will cause the blood pressure to be slightly lower than when the client is standing.

The nurse is assessing the characteristics and positioning of the client's uvula, which deviates asymmetrically when the nurse has the client say "aaah." This finding should prompt the nurse to focus on which of the following during subsequent assessment?

The client's neurological status Explanation: Deviation of the uvula or lack of movement of the soft palate suggests cranial nerve damage or stroke. Further neurological assessment and referral is necessary. This abnormal finding is not associated with immune, respiratory, or nutritional deficits.

The nurse begins the physical examination of a newly admitted client by assessing the client's mental status. What is the nurse's best rationale for performing the mental status exam early in the assessment?

The exam can provide clues about the validity of the client's responses now and throughout. Explanation: Assessing mental status at the very beginning of the head-to-toe examination provides clues regarding the validity of the subjective information provided by the client during the history and throughout the exam. Thus, it is best to determine the validity of client responses before completing the entire physical exam only to learn that the client's answers to questions may have been inaccurate. Assessing mental status first will not necessarily lessen a client's anxiety or fears about a serious illness. The exam can provide data about mental health problems. However, this is not the primary reason for performing the exam at the very beginning.

Which illustrates the nurse using the technique of inspection?

The nurse detects a fruity odor of the client's breath. Explanation: Inspection involves conscious observation of the client's physical characteristics and behaviors and smelling for odors. The nurse uses the technique of inspection to detect a fruity odor to the client's breath. The nurse uses the technique of palpation to note increased warmth surrounding an incision. Auscultation is used by the nurse to assess the lub-dub sounds of the heart. The nurse detects tympanic sounds of the bowel by percussing the abdomen.

Which describes the nurse using the technique of auscultation?

The nurse detects gurgling throughout the abdomen. Explanation: Auscultation is used by the nurse to assess bowel sounds, such as gurgling throughout the abdomen. Inspection involves conscious observation of the client's physical characteristics and behaviors and smelling for odors, such as foul smelling urine. The nurse uses palpation to detect nodules in the breast by the use of touch. The nurse uses the technique of percussion to produce sounds over various parts of the body, such as dullness over the liver.

Which is an example of inspection? Select all that apply.

The nurse notes a fine rash covering the individual's thorax. The nurse notes symmetry of the individual's thorax. The nurse detects foul odor of the urine. Explanation: Inspection involves conscious observation of the client's physical characteristics and behaviors and smelling for odors. The nurse uses the technique of inspection to detect a foul odor to the client's urine and note rashes and symmetry of the thorax. The nurse uses the technique of palpation to detect masses. Auscultation is used by the nurse to assess lung sounds, such as crackling.

A client reports after a back massage that his lower back pain has decreased from 8 to 3 on the pain scale. What opioid neuromodulator may be responsible for this increased level of comfort?

The release of endorphins Explanation: Endorphins and enkephalins are opioid neuromodulators that are powerful pain-blocking chemicals that have prolonged analgesic effects and produce euphoria. It is thought that certain measures such as skin stimulation and relaxation techniques release endorphins.

A nurse practitioner is assessing the tympanic membrane of a client who has come to the clinic. What would the nurse practitioner expect to visualize if the client has a normal otoscopic evaluation?

The short process of the malleus Explanation: During visualization of the normal tympanic membrane, it is intact and translucent and the short process of the malleus is visible. The nurse practitioner would not expect to see the stapes or the head of the incus.

A nurse is preparing a teaching session for a group of new parents about ear infections and measures to prevent them. The nurse is planning to address the reasons why children are more susceptible to these infections than adults. Which information would the nurse describe?

The size and shape of children's eustachian tubes makes them vulnerable. Explanation: The fact that children are more susceptible than adults to otitis media is due mostly to the shorter, straighter, narrower eustachian tubes of children. Otitis media in children is not normally associated with putting things in their ears, immature immune systems, or poor hygiene.

When examining the head, the nurse remembers that the anatomic regions of the cranium take their names from which of the following sources?

The underlying bones Explanation: Regions of the head take their names from the underlying bones of the skull, not from the names of anatomists, anatomical positions, or vasculature.

Palpation of a 15-year-old boy's submandibular lymph nodes reveals them to be enlarged and tender. What is the nurse's most reasonable interpretation of this assessment finding?

There is an infection in the area that these nodes drain. Explanation: Whenever a lymph node is enlarged or tender, the nurse should assess for infection in the area that the particular nodes drain. Thyroid or muscular involvement is less likely, and infection does not likely underlie the nodes directly.

The nurse is providing health education to an elderly client with dysphagia following a recent ischemic stroke. What would be most appropriate for the nurse to include?

Thoroughly chew small amounts of food with each mouthful. Explanation: Dysphagia, difficulty swallowing, increases the risk of aspiration. Thoroughly chewing small bites of food decreases this risk and is most critical for safety. Fully raising the head of the bed prevents aspiration. Dysphagia is not associated with temporomandibular joint pain, and the client may drink during meals unless explicitly contraindicated.

A 58-year-old man who is HIV-positive has presented with thick, white plaques on his oral mucosa. What diagnosis would the nurse first suspect?

Thrush Explanation: Thick, white plaques that are partially adherent to the oral mucosa are associated with thrush. HIV and AIDS are predisposing factors. People with HIV and AIDS are also prone to Kaposi's sarcoma, but these lesions are typically deep purple. Diphtheria causes a dull redness in the throat, and a torus palatinus is a bony growth in the hard palate.

A nurse wants to assess a client's orientation. The nurse recognizes that which orientation is usually lost first when the client is confused?

Time Explanation: Orientation to time is usually lost first and orientation to person is lost last.

A nurse is interviewing a client regarding her lifestyle and health practices to obtain subjective information to assist in her assessment of her skin. She asks her, "Do you spend long periods of time sitting or lying in one position?" Which of the following is the best rationale for asking this question?

To determine the clients risk for pressure ulcers Explanation: Older, disabled, or immobile clients who spend long periods of time in one position are at risk for pressure ulcers. Spending long periods of time sitting or lying in one position is not associated with increased risk for skin cancer, dehydration, or herpes zoster.

While performing the physical examination of a client, a nurse lightly taps certain parts of the body to produce sound waves. What is the purpose of this method of assessment?

To determine whether a structure is filled with air or fluid or is a solid structure Explanation: The nurse uses the percussion technique while performing a physical examination to determine whether the underlying structure is filled with air or fluid or is a solid structure. Palpation technique is used to feel deep organs or structures covered by thick muscles and to determine tenderness, moisture, and surface skin texture. The nurse uses the inspection technique to look for abnormalities on the skin's surface.

What is the primary purpose of health assessment?

To gather information about the health status of the client Explanation: Health assessment is "gathering information about the health status of the client, analyzing and synthesizing those data, making judgments about nursing interventions based on the findings, and evaluating client care outcomes" (AACN, 2011). Health assessment is not making judgments about a client's lifestyle; it does not involve care based on the nurse's views and beliefs, nor does it help the physician diagnose illness without further testing.

During the comprehensive health assessment, the nurse asks several questions relating to the client's family history of illnesses, such as diabetes and cancer. Why does the nurse do this? Select all that apply.

To help identify those diseases for which the client may be at risk To provide counseling and health teaching in high-risk areas To identify genetic family trends for which the client is at risk Explanation: The nurse asks the client about the health of close family members (i.e., parents, grandparents, siblings) to help identify those diseases for which the client may be at risk and to provide counseling and health teaching. Information concerning client and family history may be elicited to identify genetic family trends. The primary reasons are not to identify a negative family history or help the client feel at ease and not worry about being sick.

The nursing instructor explains that sometimes a nurse uses a mnemonic, such as OLDCARTS, as the nurse completes the assessment. What is the purpose of the mnemonic?

To remember the elements that are important to assess with a symptom Explanation: OLDCARTS is one example and stands for onset, location, duration, character, associated/aggravating factors, relieving factors, timing, and severity. OLDCARTS does not help a nurse remember the parts of a focused assessment, order of assessment, or how to document findings.

The wife of a client with cancer is concerned that her husband's breakthrough doses of morphine have recently needed to be larger and more frequent in order for him to achieve pain relief. The nurse would recognize that the client is likely showing the effects of which of the following?

Tolerance Explanation: This client is likely developing drug tolerance, which occurs when the body becomes accustomed to the opioid and needs a larger dose each time for pain relief. This is not a pathological finding and does not necessarily indicate physical dependence. Tolerance does not indicate addiction or a heightened risk of addiction. The phenomenon noted is not indicative of a drug interaction.

A young man is concerned about a hard mass in the midline of his palate that he has just noticed. Examination reveals that it is indeed hard and in the midline. No mucosal abnormalities are associated with this lesion. The client has no other symptoms. What is the most likely diagnosis?

Torus palatinus Explanation: Torus palatinus is relatively common and benign but can go unnoticed by clients for many years. The appearance of a bony mass can be concerning. Leukoplakia is a white lesion on the mucosal surfaces corresponding to chronic mechanical or chemical irritation. It can be premalignant. Thrush is usually painful and seen in immunosuppressed clients or those taking inhaled steroids for COPD or asthma. Kaposi's sarcoma is usually seen in HIV-positive people; these lesions are classically deep purple.

A client seeks medical attention for sharp, shooting facial pain that lasts for several minutes at a time. For which health problem should the nurse assess this client?

Trigeminal neuralgia Explanation: Trigeminal neuralgia is manifested by sharp, shooting, piercing facial pain that lasts from seconds to minutes. The pain occurs over the divisions of the fifth trigeminal cranial nerve. A headache associated with a fever or high blood pressure is a cluster headache. Tension headaches are caused by tightening of facial and neck muscles. Migraine headaches are provoked by hormone fluctuations.

A parent is very upset because she is told her child has a refractive error. The nurse reassures the parent that refractive errors are the most common visual change in children.

True

The nurse finds no adequate medical or physical explanation for the symptoms a client is experiencing. This would be considered a somatoform symptom. (True of False)

True

When assessing the gastrointestinal system, the nurse correctly asks, "Do you have any trouble swallowing?"

True

What is the most important focus area for the integumentary system?

UV radiation exposure Explanation: Excessive UV radiation is the most important focus area for the integumentary system, because exposure to it has been shown to cause skin cancers, particularly melanoma. Chemical exposure, moles with defined borders smaller than 6 mm, and hygiene of the face and hands are not the most important focus areas for the integumentary system.`

How should the nurse place the ear of an adult when using the otoscope?

Up and back

Mrs. Anderson presents with an itchy raised rash that appears and disappears in various locations. Each lesion lasts for many minutes. Which most likely accounts for this rash?

Urticaria or hives Explanation: This is a typical case of urticaria. The most unusual aspect of this condition is that the lesions move from place to place. This would be distinctly unusual for the other causes listed.

How can a nurse accurately assess the distant visual acuity of a client who is non-English speaking?

Use a Snellen E chart to perform the examination Explanation: If a client does not speak English, is unable to read, or has a verbal communication problem, the Snellen E chart can be used to test the client's distant visual acuity. With this test, the client is asked to indicate by pointing which way the E is open on the chart. The six cardinal positions of gaze test eye muscle function and cranial nerve function. The Jaeger chart tests near visual acuity. Confrontation test is used to test visual fields for peripheral vision.

The nurse is the primary care provider for a 21-year-old man who, as the result of a brain injury suffered in a mountain-biking accident in his teens, has the cognitive abilities of a 9-year-old. How should the nurse accommodate the client's cognition and comprehension during assessment?

Use the client's family as a source of information. Explanation: Limitations on intelligence often require the clinician to use the client's family as a source of assessment data, though it is still appropriate to direct questions to the client himself. It would be simplistic to downplay the interview and rely solely on the written history or to categorically reject subjective assessment.

A nurse is providing care to a client who has been in a motor vehicle accident and who has facial lacerations and a pelvic fracture. How can the nurse best determine the reliability and accuracy of data obtained during a pain assessment?

Validate the assessment data with the client. Explanation: It is important to validate pain assessment data that are collected with the client. Comparisons to previous assessments and consultation with other members of the care team do not determine the reliability and accuracy of data obtained during a pain assessment.

A client reports, "I feel like the whole room is spinning around me, and it makes me nauseous sometimes." What term should the nurse use to document the client's symptom?

Vertigo Explanation: Vertigo, the sensation of the room spinning, indicates dysfunction of the bony labyrinth in the inner ear. Dizziness indicates that the client feels like he/she is spinning in the room. Tinnitus, a sensation of buzzing in the ear, is thought to be an inability to filter internal noise from the external input of sound. Otalgia is ear pain.

When visualizing the structures of the nose, the nurse recalls that air travels from the anterior nares to the trachea through the:

Vestibule, nasal passages, and nasopharynx Explanation: After entering the anterior nares, air enters the vestibule and passes through the narrow nasal passage to the nasopharynx.

As part of a mental status assessment, the nurse asks a client to draw the face of a clock. The nurse is assessing which of the following?

Visual perceptual and constructional ability Explanation: Asking a client to draw the face of a clock tests the client's visual perceptual and constructional ability. Concentration is evaluated by noting the client's ability to focus and stay attentive. Orientation is tested by asking the client to state his or her name and the names of family members, time, day or season, and place. Thought processes and perceptions are evaluated by asking the client to say more about or verbalize his or her understanding of the current situation. Expressions and feelings are evaluated by asking the client how he or she is feeling and about plans for the future.

A client is found to have a smooth, glossy tongue. What vitamin deficiency might this indicate?

Vitamin B12 deficiency Explanation: The tongue and buccal mucosa may appear smoother and shiny from papillary atrophy and thinning of the buccal mucosa. This condition is called smooth glossy tongue and may result from deficiencies of riboflavin, folic acid, and vitamin B 12.

The nurse notes that the client's tongue appears as shown. What should the nurse suspect is occurring with this client?

Vitamin deficiency Explanation: A smooth and often sore tongue that has lost its papillae, sometimes just in patches, suggests a deficiency in riboflavin, niacin, folic acid, vitamin B12, pyridoxine, or iron. Candidiasis or white patches would be on the tongue if a yeast infection is present. The tongue may have black areas if antibiotics were recently used. The tongue would deviate from the midline if CN XII is damaged.

The nurse is assessing a dark-skinned client whose forearms and hands have distinct regions of depigmentation. The nurse should document the presence of what health problem?

Vitiligo Explanation: Vitiligo is characterized by discrete areas of depigmentation. Albinism is a generalized absence of pigment, and striae are often known as stretch marks. Angiomas are small, raised skin lesions.

The nurse is assessing a client's respiratory rate. Which of the following should the nurse do to ensure accuracy of this assessment?

Watch chest movement before removing the stethoscope after counting the apical beat Explanation: Because breathing is under voluntary in addition to autonomic control, clients may intentionally or inadvertently alter their breathing rate if they are aware that it is being assessed. To obtain an accurate assessment, observe respirations without alerting the client by watching chest movement before removing the stethoscope after you have completed counting the apical beat. Asking the client to breathe normally may still make the client self-conscious and prevent an accurate measurement. Observing the clients chest movement before calling the client back to the examination room would not be practical due to the distance. Performing the assessment multiple times is unnecessary and time consuming

The nurse is caring for the client who is receiving heparin. The nurse plans to:

Wear clean gloves when administering heparin to the client Explanation: Heparin is an anticoagulant administered subcutaneously in the abdomen, which may expose the nurse to direct contact with the client's body fluids. The nurse wears clean gloves when administering heparin and after administering the heparin does not recap the needle and performs hand hygiene with alcohol-based gel. A mask is not required when administering heparin to the client.

A nurse is admitting a new client to the subacute medical unit and is completing a comprehensive assessment. The nurse is appropriately applying standard precautions by performing what action?

Wearing gloves to palpate the tongue and buccal membranes Explanation: When adhering to standard precautions, the nurse would wear gloves when it can be reasonably anticipated that contact with blood or other potentially infectious materials, mucous membranes, nonintact skin, or potentially contaminated intact skin (e.g., of a client incontinent of stool or urine) could occur. Safety pins should be disposed of in the sharps container. Gowns and masks are appropriate only if anticipated client interaction indicates that contact with blood or body fluids may occur. Hand hygiene need not be performed between assessments of each system or body part.

Which action by a nurse demonstrates the correct application of the principles of standard precautions?

Wearing gloves when palpating the tongue, lips, & gums Explanation: The nurse should wear gloves when examining or touching any areas where there is the potential for exposure to blood or body fluids. Gloves are changed between tasks and procedures on the same client after contact with material that may contain a high concentration of microorganisms. Wearing a gown, gloves, and mask is not necessary for the entire physical assessment. If hands are visibly soiled, the nurse should wash with soap and water.

The nurse notes that an older client speaks rapidly and uses words that make no sense or communicate any clear meaning. When documenting this finding, the nurse should use which term to describe this client's speech?

Wernicke's aphasia Explanation: Wernicke's aphasia is rapid speech that lacks meaning. It is caused by a lesion in the posterior superior temporal lobe. Dysphonia is a voice volume disorder, caused by an issue within the larynx or impairment of cranial nerve X. Dysarthria is a defect in the muscles that control speech. Cerebellar dysarthria is irregular uncoordinated speech caused by multiple sclerosis.

The submandibular glands open under the tongue through openings called

Wharton ducts. Explanation: The submandibular glands, located in the lower jaw, open under the tongue on either side of the frenulum through openings called Wharton's ducts.

The three salivary glands also contain drainage ducts. Which drainage ducts are associated with the submandibular gland?

Wharton's ducts Explanation: The submandibular gland is beneath the body of the mandible. Its Wharton ducts run deep to the floor of the mouth and open on both sides of the frenulum. The parotid (Stensen's) duct opens into the mouth in the buccal mucosa just opposite the upper second molar. The small sublingual salivary gland lies within the floor of the mouth under the tongue with many openings along the submandibular duct.

The nurse is conducting a MMSE (Mini Mental State Examination) on a 77-year-old woman brought to the emergency department by her daughter. What is one of the first questions the nurse would ask this client?

What day of the week is it? Explanation: Short- and long-term memories indicate the current level of cognitive functioning. Altered memory may be from dementia, Alzheimer disease, or other disorders.

The nurse enters an older client's room to assess for pain and discovers the client is hard of hearing. What is the nurse's best action?

When assessing the older client for pain, determine whether the client has any auditory impairment. If so, position your face in the client's view, speak in a slow, normal tone of voice, reduce extraneous noises, and provide written instructions. The FLACC scale is used primarily for infants. Hearing aids are expensive and suggesting to purchased one does not aid in the pain assessment at present.

Student nurses are practicing taking comprehensive health histories from one another. What components should be included in a comprehensive health history? (Select all that apply.)

When coughing began Pain location Pain duration Pain intensity Explanation: Location, duration, intensity, description, aggravating factors, alleviating factors, and functional impairment are components of a comprehensive health history. Asking the client what was eaten for the last three meals does not fall into the definition of a comprehensive health history

What light should the nurse use to inspect a lesion on the thigh of a client for the presence of fungus?

Wood's light Explanation: The nurse should inspect the lesion under Wood's light to confirm the presence of fungus on the lesion. Wood's light is an ultraviolet light filtered through a special glass that shows a blue-green fluorescence if the lesion is due to fungal infection. The lesion can be inspected in sunlight and artificial light, but it may not indicate the type of infection in the lesion. Lesions cannot be inspected properly using a flashlight.

A nurse is examining a client suspected of having a fungal infection of the skin. Which piece of equipment should the nurse use to confirm the presence of fungus?

Wood's light Explanation: Wood's light is a special piece of examination equipment that is used to test for fungus. A penlight is used for more general assessments, such as of the mouth and throat and to transilluminate the sinuses. A magnifying glass is used to enlarge the visibility of a lesion. An examination light is used to better illuminate the client's body as a whole to facilitate physical examination.

A decrease in tongue strength is noted on examination of a client. The nurse interprets this as indicating a problem with which cranial nerve?

XII Explanation: Decreased tongue strength may occur with a defect of the twelfth cranial nerve. The third cranial nerve is involved with eye muscle movement. The sixth cranial nerve is involved with lateral eye movement. The eighth cranial nerve is involved with hearing and equilibrium

When answering questions about health during a presentation at a women's club luncheon, the nurse emphasizes that prevention of disease is multifaceted but is connected directly to

a healthy lifestyle Explanation: Education is key to assisting people in making the connection between a healthy lifestyle and the prevention of disease. The absence of disease does not define health. Stress reduction and changes in the environment influence a person's ability to live up to his or her potential as it relates to health.

After examining the client's tympanic membranes, the nurse documents "Right tympanic membrane, red and bulging with no light reflex." The nurse recognizes that these are signs of

acute otitis media. Explanation: In acute otitis media there is a bulging red membrane with decreased or absent light reflex.

A past history is being taken by the nurse for a client with COPD. The nurse includes which elements in this part of the health history? Select all that apply.

allergies childhood illnesses health maintenance

The meibomian glands secrete

an oily substance to lubricate the eyes. Explanation: Meibomian glands secrete an oily substance that lubricates the eyelid.

The CAGE assessment is used by the nurse to determine if further assessment is needed. The nurse may assess that it is highly likely the client has a problem and would seek additional assessments if the client

answered "yes" to three of the four CAGE questions. Explanation: The CAGE assessment is a quick questionnaire used to determine if an alcohol assessment is needed. If two or more of these questions is answered yes, then further assessment is advised.

The apocrine glands are dormant until puberty and are concentrated in the axillae, the perineum, and the

areola of the breast. Explanation: The apocrine glands are associated with hair follicles in the axillae, perineum, and areola of the breast. Apocrine glands are small and non-functional until puberty at which time they are activated and secrete a milky sweat.

When the nurse asks the client to explain similarities and differences between objects, what cognitive ability is being tested?

bstract reasoning. Explanation: Abstract reasoning is the ability to compare objects. For example, "How are an apple and orange the same? How are they different?" Also, asking to explain a proverb. For example, "A rolling stone gathers no moss" or "A stitch in time saves nine."

While conducting an assessment the nurse suspects that a client is making up things in response to specific questions. What behavior is this client demonstrating?

confabulation Explanation: Confabulation is the fabrication of facts or events in response to questions in order to fill in the gaps from impaired memory. Derailment is tangential speech with shifting topics that are loosely connected or unrelated. Perseveration is persistent repetition of words or ideas. Flight of ideas is an almost continuous flow of accelerated speech with abrupt changes from one topic to the next.

An adult client tells the nurse that his eyes are painful because he left his contact lenses in too long the day before yesterday. The nurse should instruct the client that prolonged wearing of contact lenses can lead to

corneal damage. Explanation: Improper cleaning or prolonged wearing of contact lenses can lead to infection and corneal damage.

A pathophysiology instructor is discussing pain and its treatment across cultures. The instructor points out that clients from racial and ethnic minorities often receive less pain medication compared to Caucasians for what specific conditions?

cute pain in the ED Explanation: African Americans, Hispanic Americans, and other clients of racial and ethnic minority heritage receive less pain medication compared to Caucasians across a range of conditions, including cancer pain, acute postoperative pain, chest pain, acute pain presenting in the ED, and chronic low back pain. This disparity may be the result of client variables such as nociceptive differences, communication processes, or pain behaviors.

An elderly client is seen by the nurse in the neighborhood clinic. The nurse observes that the client is dressed in several layers of clothing, although the temperature is warm outside. The nurse suspects that the client's cold intolerance is a result of

decreased body metabolism. Explanation: Research has shown that for older adults, normal body temperature values for all routes are consistently lower than values reported in younger populations.

During a comprehensive assessment the client reports having a college education, is employed as a human resource director, and is 45 years old. Which part of the assessment should the nurse document this information?

dentifying data Explanation: Education, employment and age are all documented within the identifying data section of the comprehensive assessment. The family history outlines the age and cause of death, of siblings, parents, grandparents, and children and includes the presence or absence of specific illnesses in the family. The chief complaint is the reason for the person seeking care. The history of present illness describes how each symptom developed. It includes the client's thoughts and feelings about the illness, relevant parts of the review of systems, and medications, allergies, and lifestyle habits that impact the present illness.

The client's daughter asks the nurse why the nurse is asking her mother depression-related questions. The nurse explains that even though the client has symptoms of dementia, the Geriatric Depression Scale is being used because

depression often mimics signs and symptoms of dementia. Explanation: The Geriatric Depression Scale is used if depression is suspected in the older client. Read the questions to the client if the client cannot read.

Hair follicles, sebaceous glands, and sweat glands originate from the

dermis. Explanation: The dermis is a well-vascularized, connective tissue layer containing collagen and elastic fibers, nerve endings, and lymph vessels. It is also the origin of sebaceous glands, sweat glands, and hair follicles.

One of the body's normal physiologic responses to pain is

diaphoresis. Explanation: Diaphoresis is associated with acute pain.

The nurse has completed an assessment on a new client. After gathering the data, formulating a nursing diagnosis, and developing a plan of care, it is important for the nurse, before finalizing the plan, to

discuss the plan with the client Explanation: Sharing the assessment and plan with the client will allow the client to offer his or her opinion, concerns, and willingness to proceed with the interventions. This makes the client an active participant in his or her plan of care.

Before examining the mouth of an adult client, the nurse should first

don clean gloves for the procedure. Explanation: Before touching any mucous membranes the nurse should apply gloves.

During examination of the oral cavity, the nurse examines the salivary glands. Which area of the mouth should the nurse assess to inspect for the Wharton's ducts?

either side of the frenulum on the floor of the mouth Explanation: The nurse should inspect the Wharton's duct on either side of the frenulum on the floor of the mouth. Stenson's ducts, not Wharton's ducts, are visible on the buccal mucosa across from the second upper molars. The right sides of the frenulum at the base of the gums and on the posterior aspect of the tongue bilaterally are not appropriate to inspect salivary ducts.

The nurse observes an inward turning of the lower lid in a 77-year-old client. The nurse documents

entropion

While assessing the eye of an adult client, the nurse observes an inward turning of the client's left eye. The nurse should document the client's

esotropia. Explanation: Esotropia is an inward turn of the eye.

A female client visits the clinic and complains to the nurse that her skin feels "dry." The nurse should instruct the client that skin elasticity is related to adequate

fluid intake. Explanation: Adequate fluid intake is required to maintain skin elasticity.

A client comes to the community clinic seeking help for acute low back pain. Which type of assessment should the nurse complete for this client?

focused Explanation: A focused assessment gathers information about the current health problem. A follow-up assessment evaluates a specific problem after treatment. An emergency assessment focuses on data to quickly resolve the immediate health problem. A comprehensive assessment includes demographic data, a full description of the reason for seeking care, individual health history, family history, functional status, and a history in all physical and psychosocial areas.

A nurse is performing a client assessment in an urgent care clinic. The most likely tool being used is the

focused assessment

The tongue is attached to the hyoid bone and styloid process of the temporal bone and is connected to the floor of the mouth by the

frenulum. Explanation: The tongue is a mass of muscle, attached to the hyoid bone and styloid process of the temporal bone. It is connected to the floor of the mouth by a fold of tissue called the frenulum.

An adult client tells the nurse that her peripheral vision is not what it used to be and she has a blind spot in her left eye. The nurse should refer the client for evaluation of possible

glaucoma. Explanation: A scotoma is a blind spot that is surrounded by either normal or slightly diminished peripheral vision. It may be from glaucoma.

The nurse is assessing a client's range of motion. Which equipment should the nurse use to validate the degrees of joint mobility?

goniometer Explanation: A goniometer is used to measure degree of flexion and extension of joints. A speculum is used to examine the ear canals. Test tubes are used to measure temperature sensation. A stadiometer is used to measure height.

The nurse is planning to assess a client's near vision. Which technique should be used?

have the client read newspaper print held 14 inches from the eyes Explanation: Near vision is tested by asking the client to read newspaper print held 14 inches from the eyes. Shining a light on the bridge of the nose tests the corneal light reflex. Moving the eyes in the direction of a moving finger tests for extraocular movements. Having the client read letters on a wall chart tests for central and distance vision.

The nurse is beginning the review of systems with a client. Which approach would ensure that all major body systems are included in this assessment?

head to toe Explanation: Reviewing the body systems from head to toe is one way to ensure that all areas are included. Following a circle, right to left, or alphabetical are not identified as patterns to complete the review of systems.

A pregnant woman comes to the physician's office for her first prenatal visit. The nurse knows the importance of performing a comprehensive health history in this case and understands the following must be included (check all that apply):

information about current pregnancy previous pregnancies obstetrical and gynecological history family history Explanation: It is important on the first prenatal visit to perform a comprehensive health history, which includes information about the present pregnancy, previous pregnancies, obstetrical and gynecological history, the family, and psychosocial profile. Food preferences are part of a nutritional assessment and not necessary at this time.

During a comprehensive assessment, the primary technique used by the nurse throughout the examination is

inspection. Explanation: Inspection involves using the senses of vision, smell, and hearing to observe and detect any normal or abnormal findings. This technique is used from the moment that you meet the client and continues throughout the examination. Inspection precedes palpation, percussion, and auscultation because the latter techniques can potentially alter the appearance of what is being inspected.

Pressure ulcers are staged as I through IV. Put the following in order from stage I through stage IV.

intact, firm skin with redness ulceration involving the dermis full-thickness skin loss necrosis with damage to underlying muscle

The nurse is preparing to assess an adult client in the clinic. The nurse observes that the client is wearing lightweight clothing that is worn and soiled, although the temperature is below freezing outside. The nurse anticipates that the client may be

lacking adequate finances. Explanation: When you meet the client for the first time, observe any significant abnormalities in the client's skin color, dress, hygiene, posture and gait, physical development, body build, apparent age, and gender. If you observe abnormalities, you may need to perform an in-depth assessment of the body area that appears to be affected.

An adult client visits the clinic and tells the nurse that he has had excessive tearing in his left eye. The nurse should assess the client's eye for

lacrimal obstruction. Explanation: Excessive tearing (epiphora) is caused by exposure to irritants or obstruction of the lacrimal apparatus. Unilateral epiphora is often associated with foreign body or obstruction.

A client with acute onset of shoulder pain is answering questions during a health history. The nurse is utilizing a mnemonic specific to the attributes of a symptom. The nurse first asks about the onset of symptoms followed by

location duration characteristic symptoms associated manifestations relieving/exacerbating factors treatment

An older client asks why vision is not as sharp as it used to be when the eyes are focused forward. What should the nurse realize this client is describing?

macular degeneration Explanation: Macular degeneration causes a loss of central vision. Risk factors for macular degeneration are age, smoking history, obesity, family history, and female gender. Cataracts are characterized by cloudiness of the eye lenses. Glaucoma is an increase in intraocular pressure that places pressure on eye structures and affecting vision. A detached retina is the sudden loss of vision in one eye. This health problem may be precipitated by the appearance of blind spots.

When documenting that a client has freckles, the appropriate term to use is

macules Explanation: Macules are skin discolorations that are flat, circumscribed, discolored, and less than 1 cm in diameter. An example of a macule is a freckle.

While assessing an adult client, the nurse observes freckles on the client's face. The nurse should document the presence of

macules. Explanation: Freckles are flat, small macules of pigment that appear following sun exposure.

The nurse is preparing to examine the sinuses of an adult client. After examining the frontal sinuses, the nurse should proceed to examine the

maxillary sinuses. Explanation: The frontal sinuses (above the eyes) and the maxillary sinuses (in the upper jaw) are accessible to examination by the nurse.

A client says that food is not important and meals are not enjoyable. Where should the nurse document this information?

nutrition health pattern Explanation: Information about diet and intake should be documented within the nutrition health pattern. There is no evidence to support that this is the a past or present health problem for the client. It would not be appropriate to document this information within the gastrointestinal review of systems.

The optic nerves from each eyeball cross at the

optic chiasma. Explanation: At the point where the optic nerves from each eyeball cross—the optic chiasma—the nerve fibers from the nasal quadrant of each retina (from both temporal visual fields) cross over to the opposite side.

Which of the following is an average normal temperature in centigrade for a healthy adult?

oral: 37.0°C Explanation: The normal range for an oral temperature is 37.0°C, a rectal temperature is 37.5°C, an axillary temperature is 36.5°C, and a tympanic temperature is 37.5°C.

The current blood pressure measurement on a 24-hour uncomplicated postoperative client while standing at the bedside is 105/65. The last two readings were 130/75 and 125/70 while resting in bed. The nurse should be alert for signs of:

orthostatic hypotension.

The nurse selects a tuning fork to use when assessing a client. Which body system is the nurse most likely assessing?

peripheral vascular Explanation: A tuning fork has two uses in the physical examination. The most common is to assess hearing however the tuning fork is also used to assess the sense of vibration when completing the neurologic or peripheral vascular assessment. A tuning fork is not used to assess the respiratory, genitourinary or gastrointestinal systems.

The nurse is preparing to examine an adult client's eyes, using a Snellen chart. The nurse should

position the client 609.6 cm (20 ft) away from the chart. Explanation: Used to test distant visual acuity, the Snellen chart consists of lines of different letters stacked one above the other. The letters are large at the top and decrease in size from top to bottom. The chart is placed on a wall or door at eye level in a well-lighted area. The client stands 20 feet from the chart and covers one eye with an opaque card (which prevents the client from peeking through the fingers). Then the client reads each line of letters until he or she can no longer distinguish them.

A client comes to the emergency department with severe abdominal pain. When performing a complete assessment, the nurse would focus on which of the following areas when covering past health history?

previous medical and surgical problems Explanation: The past health history includes asking about previous medical and surgical problems along with their dates. Aggravating factors, duration, and intensity of the pain are all part of the history of present illness.

During a health class, the nurse is emphasizing exercise and healthy eating. The level of prevention being utilized by the nurse is

primary prevention Explanation: Exercise and healthy eating improve wellness and help protect from disease and disability, which is primary prevention.

The nurse notes that a client's nails are greater than a 160-degree angle. What should the nurse assess as a priority for this client?

pulse oximetry Explanation: A nail angle greater than 160 degrees indicates clubbing which is caused by chronic hypoxia. Measuring the client's pulse oximetry would be a priority. Heart sounds, bowel sounds, and body temperature will not provide information to determine the cause for the clubbed nails.

When doing a complete health history, the nurse usually collects demographical data first and a reason for seeking care. What additional data the nurse collects depends upon (check all that apply):

reason for the visit pertinence of the data time restrictions within the setting Explanation: The nurse determines what data to collect beyond the minimum required and bases this decision on the reason for the visit, pertinence of the data, and time restrictions within the setting. The time of day and the client's diet have nothing to do with the information collected in the health history.

The nurse assesses the client's vital signs as follows: respirations 20 breaths/minute, tympanic temperature 100.9°F, pulse 88 beats/minute, and blood pressure 104/64 mm Hg. The nurse should

record the vital signs. Explanation: Validate the assessment data you have collected. This is necessary to verify that the data are reliable and accurate. Document the assessment data following the health care facility or agency policy.

A client is experiencing weakness of the left side of the body. Which piece of equipment should the nurse use to determine if the client's neurologic system is intact?

reflex hammer Explanation: A reflex is used to assess deep tendon reflexes which are under the control of the neurologic system. A penlight is used to assess pupillary reflexes and aids with tangential lighting. A scoliometer measures the degree of spinal curvature. A pulse oximeter measures oxygen level.

Photoreceptors of the eye are located in the eye's

retina. Explanation: The innermost layer, the retina, extends only to the ciliary body anteriorly. It receives visual stimuli and sends it to the brain. The retina consists of numerous layers of nerve cells, including the cells commonly called rods and cones. These specialized nerve cells are often referred to as "photoreceptors" because they are responsive to light

A client tells the nurse that he is constantly congested in spite of using a decongestant at least twice daily. The nurse understands the client may be experiencing

rhinitis medicamentosa

The nurse understands that health promotion is a very important part of nursing care. When performing the health history, there are many different opportunities for the nurse to teach healthy behaviors. One way the nurse can do this is by focusing on which of the following topics:

sexual history and pattern Explanation: There are many opportunities for the nurse to promote healthy behaviors. When assessing high-risk clients with multiple partners, the nurse can seize this opportunity to provide information that can prevent disease and illness. Gender, culture, and spirituality are not generally factors in teaching about health promotion.

The nurse notes that an older adult client is wearing multiple layers of clothing on a warm fall day. What would be the nurse's priority assessment at this time?

sking whether the client often feels cold Explanation: Dress typically is appropriate for occasion and weather. Dress varies considerably from person to person. Some older adults may wear excess clothing because of slowed metabolism and loss of subcutaneous fat, resulting in cold intolerance. The nurse needs to determine this first before performing any other assessments.

The nurse assesses an older adult bedridden client in her home. While assessing the client's buttocks, the nurse observes that a small area of the skin is broken and resembles an erosion. The nurse should document the client's pressure ulcer as

stage II. Explanation: Stage II pressure ulcer is a partial thickness loss of dermis presenting as a shallow open ulcer with a red-pink wound bed, without slough. May also present as an intact or open/ruptured, serum-filled blister. Presents as a shiny or dry shallow ulcer without slough or bruising; bruising indicates suspected deep tissue injury. This stage should not be used to describe skin tears, tape burns, perineal dermatitis, maceration, or excoriation.

A client comes to the ED complaining of chest pain. This would be considered

subjective primary data Explanation: The individual client is considered the primary data source. When possible, clients provide subjective information regarding their health behaviors and situations. Subjective information is from the perspective of the client.

An adult male client visits the outpatient center and tells the nurse that he has been experiencing patchy hair loss. The nurse should further assess the client for:

symptoms of stress. Explanation: Patchy hair loss may accompany infections, stress, hairstyles that put stress on hair roots, and some types of chemotherapy.

The nurse understands that when performing a health history it is important to include subjective data, which includes:

symptoms that the client reports Explanation: Subjective data are based on the signs and symptoms that the client reports. They may not be perceived by the observer and are not taken from values of lab reports or vital signs.

A client complains of a headache over both temporal areas. What type of headache should the nurse suspect the client is experiencing?

tension Explanation: Tension headaches often arise in the temporal areas. Cluster headaches typically occur behind the eyes. A throbbing, severe, unilateral headache that lasts 6-24 hours and is associated with photophobia, nausea, and vomiting suggests a migraine headache. Hypertensive is not a type of headache although individuals with hypertension may experience a headache upon arising in the morning.

An adult client visits the clinic complaining of a sore throat. After assessing the throat, the nurse documents the client's tonsils as 4+. The nurse should explain to the client that 4+ tonsils are present when the nurse observes tonsils that are

touching each other. Explanation: 4+ Tonsils touch each other.

An older client visits the clinic accompanied by his daughter. The daughter tells the nurse that her father has been experiencing severe headaches that usually begin in the morning and become worse when he coughs. The client tells the nurse that he feels dizzy when he has the headaches. The nurse refers the client for further evaluation because these symptoms are characteristic of a

tumor-related headache. Explanation: Tumor-related headaches have no prodromal stage; may be aggravated by coughing, sneezing, or sudden movements of the head.

The nurse is preparing the examination room before assessing a client. What is the purpose for a clean folded sheet on the examination table?

use as a drape Explanation: During the examination, one body part should be exposed at a time. The sheet serves as a drape to keep the other body parts covered. The sheet is not used to pad the table, collect body fluids, or to be a head support.

To assess an adult client's skin turgor, the nurse should

use two fingers to pinch the skin under the clavicle. Explanation: To assess turgor, gently pinch the skin over the clavicle with two fingers.

Short, pale, and fine hair that is present over much of the body is termed

vellus. Explanation: Vellus hair (peach fuzz) is short, pale, fine, and present over much of the body.

A client's skin color depends on melanin and carotene contained in the skin, and the

volume of blood circulating in the dermis. Explanation: The major determinant of skin color is melanin. Other significant determinants include capillary blood flow, chromophores (carotene and lycopene), and collagen.


Kaugnay na mga set ng pag-aaral

Vsim Brittany Long Complex (Pre/Post)

View Set

Hull's 16 Major Postulates (1943)

View Set

Respiratory Infections (Peds-II)

View Set

Chapter 16 Intrapartum Complications

View Set

Chapter 35: Nationalism and Political Identities in Asia, Africa, and Latin America

View Set

Chapter 1 (3 sections) Introduction to Economics

View Set

Chapter 36: Abdominal and Genitourinary Trauma

View Set

Chapter 7: Attitudes, Behavior, and Rationalization

View Set

Pharm Exam 4 CH 17 Drugs for Resp Sys

View Set